2. Pulmonology (PACKRAT 9, 11, 12, 13, 14, 15)

Ace your homework & exams now with Quizwiz!

Health Maintenance/Pulmonology A 62 year-old female is admitted to a nursing home during an outbreak of influenza. In review of her records, you note that she did not receive the flu vaccine this year. Which of the following is the most appropriate drug of choice for influenza prophylaxis in this patient? A. Ciprofloxin (Cipro) B. Oseltamivir phosphate (Tamiflu) C. Clarithromycin (Biaxin) D. Alpha-2b interferon (Avonex)

Explanations (u) A. Ciprofloxin is indicated for postexposure prophylaxis of anthrax. (c) B. Either oseltamivir or zanamivir are indicated for prophylactic use against influenza A or B. (u) C. Clarithromycin is indicated for prophylaxis against disseminated Mycobacterium avium complex. (u) D. Alpha-2b interferon is indicated for treatment of several disorders, such as chronic hepatitis B & C, but has no role in prophylactic treatment of any condition.

History & Physical/Pulmonology A 27 year-old woman presents with one week of worsening productive cough, dyspnea, fever and malaise. Which of the following physical examination findings would support the diagnosis of lobar pneumonia? A. Decreased tactile fremitus B. Egophony C. Hyperresonance to percussion D. Wheeze

Explanations (u) A. Consolidation would increase the transmission of vocal vibrations and manifest as increased tactile fremitus. (c) B. Transmission of vocal sounds through consolidation leads to the changes heard with egophony. (u) C. Dullness, not hyperresonance, would be expected with consolidation. (u) D. Wheezing is associated with narrowing of the small airways as seen in asthma.

Clinical Therapeutics/Pulmonology A 36 year-old male developed a sore throat and was treated with IM penicillin. Within 20 minutes, he felt faint and became dyspneic. Upon entry to the emergency department, he was pale and apprehensive. He had a thready pulse, and systolic blood pressure was 40 mmHg. Which of the following is the most appropriate initial agent to use? A. dopamine B. epinephrine C. hydrocortisone D. diphenhydramine

Explanations (u) A. Dopamine is not indicated in the treatment of allergic reactions. (c) B. Epinephrine is the drug of first choice for emergency use and should be given as soon as anaphylactic shock is suspected or diagnosed. (u) C. Hydrocortisone should be given as an adjunct to epinephrine, but it is not the drug of first choice. (u) D. Diphenhydramine should be given as an adjunct to treatment, but it is not the drug of first choice.

History & Physical/Pulmonology Which of the following is a common symptom associated with laryngotracheobronchitis (viral croup)? Answers A. drooling B. high fever C. "hot potato" voice D. barking cough

Explanations (u) A. Drooling and a "hot potato" voice are seen with epiglottitis, not viral croup. (u) B. Fever is usually absent or low grade in patients with viral croup. (u) C. See A for explanation. (c) D. Viral croup is characterized by history of an upper respiratory tract symptoms followed by onset of a barking cough and stridor.

History & Physical/Pulmonology On physical examination you note diminished breath sounds over the right lower lobe with decreased tactile fremitus and dullness to percussion. Which of the following is the most likely cause? A. Asthma B. Pneumonia C. Pneumothorax D. Pleural effusion

Explanations (u) A. Asthma is characterized by decreased tactile fremitus, but would have resonant to hyperresonant percussion, not dullness. (u) B. Lobar pneumonia is characterized by dullness to percussion, but would have an increased, not decreased, tactile fremitus. (u) C. A pneumothorax is characterized by decreased to absent tactile fremitus, but would have a hyperresonant percussion note, not dullness. (c) D. Decreased tactile fremitus and dullness to percussion would be found in a pleural effusion.

Clinical Therapeutics/Pulmonology A 55 year-old man with a history of chronic bronchitis presents with two days of increased dyspnea and cough with worsening purulent sputum production. He is currently using inhaled albuterol as needed. In addition to systemic corticosteroids, what pharmacologic agent is warranted at this time for treatment of this patient? A. Antibiotic B. Inhaled corticosteroid C. Long acting beta-agonist D. Theophylline

Explanations (c) A. Empiric antibiotic treatment is indicated in the treatment of acute exacerbations of COPD if there are sputum changes suggestive of bacterial infection, such as increased quantity and purulence. (u) B. Inhaled corticosteroids are not indicated in the management of acute exacerbations of COPD. (u) C. Long acting beta-agonists are not indicated in the management of acute exacerbations of COPD. (u) D. Theophylline is rarely used in the management of COPD and has no place in the management of acute exacerbations of COPD.

History & Physical/Pulmonology Kussmaul breathing is characterized by A. rapid, deep labored breathing. B. irregular and varying depth of breathing. C. frequently interspersed deeper breaths. D. periods of deep breathing alternate with periods of apnea.

Explanations (c) A. Kussmaul breathing is characterized by rapid deep labored breathing. (u) B. This characterizes ataxic breathing. (u) C. This characterizes sighing. (u) D. Periods of deep breathing alternate with periods of apnea is Cheyne-Stokes breathing.

Scientific Concepts/Pulmonology Which of the following causes of pneumonia is most likely to be complicated by diarrhea? A. Legionella B. Chlamydophila C. Mycoplasma D. Pneumococcal

Explanations (c) A. Legionella is a water-borne pathogen that can cause diarrhea as a complication of its infection. (u) B. Chlamydophila pneumonia is associated with dry cough, low-grade fever, and hoarseness. (u) C. Mycoplasma pneumonia is most likely complicated by bullous myringitis, hoarseness, rash, and chronic cough. (u) D. Pneumococcal pneumonia presents with a single rigor, rust-colored sputum, and productive cough. There is no diarrhea as part of its infection.

Diagnosis/Pulmonology Which histologic type of lung cancer is typically centrally located? A. Adenocarcinoma B. Bronchoalveolar C. Large cell D. Squamous cell

Explanations (u) A. Adenocarcinoma of the lung typically presents as a peripheral lesion. (u) B. Bronchoalveolar carcinoma, actually a subset of adenocarcinoma of the lung, typically presents as a peripheral lesion. (u) C. Large cell lung cancers usually develop as peripheral lesions. (c) D. Most squamous cell lung cancers are centrally located.

Scientific Concepts/Pulmonology Patients with long-term exposure to silica, coal dust, and asbestos may develop which of the following as complications? A. Airwayhyperreactivity B. Epithelial hyperplasia C. Pulmonary fibrosis D. Upper airway obstruction

Explanations (u) A. Airway hyperreactivity is classically seen with asthma and while acute exposure to occupational mineral dusts may cause airway hyperreactivity, this does not persist in long-term exposures. (u) B. Epithelial hyperplasia is one of the mechanisms involved in the pathogenesis of chronic bronchitis and is not involved in the pneumoconioses. (c) C. The principal processes in the pathogenesis of this set of diseases is inflammation and subsequent fibrosis. (u) D. The upper airway is not involved in this disease process.

Clinical Therapeutics/Pulmonology A 42 year-old male with unremarkable past medical history is admitted to the general medical ward with community-acquired pneumonia. He has a 20 pack-year history of cigarette smoking. He is empirically started on ceftriaxone (Rocephin). Which of the following antibiotics would be most appropriate to add to his empiric treatment regimen? Answers A. Piperacillin B. Vancomycin C. Clindamycin D. Azithromycin

Explanations (u) A. Antipseudomonal penicillins, such as piperacillin, added to an aminoglycoside or fluoroquinolone are indicated for empiric treatment of nosocomial pneumonia. (u) B. Use of vancomycin should be restricted to cases where there is a high index of suspicion of involvement of methicillin resistant Staphylococcus aureus (MRSA). (u) C. Clindamycin is indicated in cases of suspected anaerobe involvement. (c) D. Patients with community-acquired pneumonia who require hospitalization on the general medical ward are treated with an extended-spectrum beta-lactam antibiotic, such as ceftriaxone, with a macrolide, such as azithromycin. Addition of a macrolide is also recommended due to the patient's smoking history and possible involvement of Hemophilus influenzae.

Clinical Therapeutics/Pulmonology A 42 year-old male with unremarkable past medical history is admitted to the general medical ward with community- acquired pneumonia. He has a 20 pack-year history of cigarette smoking. He is empirically started on ceftriaxone (Rocephin). Which of the following antibiotics would be most appropriate to add to his empiric treatment regimen? A. piperacillin (Pipracil) B. vancomycin (Vancocin) C. clindamycin (Cleocin) D. azithromycin (Zithromax)

Explanations (u) A. Antipseudomonal penicillins, such as pipercillin, added to an aminoglycoside or fluoroquinolone are indicated for empiric treatment of nosocomial pneumonia. (u) B. Use of vancomycin should be restricted to cases where there is a high index of suspicion of involvement of methicillin resistant Staphylococcus aureus (MRSA). (u) C. Clindamycin is indicated in cases of suspected anaerobe involvement. (c) D. Patients with community-acquired pneumonia who require hospitalization on the general medical ward are treated with an extended-spectrum beta-lactam antibiotic, such as ceftriaxone, with a macrolide, such as azithromycin. Addition of a macrolide is also recommended due to the patient's smoking history and possible involvement of Hemophilus influenzae.

Health Maintenance/Pulmonology In patients with COPD, which of the following has been shown to decrease rate of malignancy and cardiovascular disease and improve survival? A. bronchodilator therapy B. pulmonary rehabilitation C. oral glucocorticosteroids D. smoking cessation

Explanations (u) A. Bronchodilator therapy is used for symptomatic treatment in patients with COPD. (u) B. Pulmonary rehabilitation improves quality of life, dyspnea and exercise capacity. It also has been shown to reduce the rate of hospitalization. (u) C. Chronic use of oral glucocorticosteroids is not recommended because of an unfavorable benefit/risk ratio. (c) D. Smoking cessation has been shown to provide significant improvement in decreasing the rate of decline in pulmonary function.

istory & Physical/Pulmonology Upon auscultation of a patient's lungs, there are harsh, hollow breath sounds which have a long inspiratory component in the region of the suprasternal notch. Throughout the periphery of the lung fields, softer breath sounds are heard. Which of the following best describes these findings? A. Normal B. Asthmatic C. Atelectasis D. Foreign body Explanations

(c) A. Bronchial breath sounds are normally heard near the sternum and vesicular breath sounds are heard over the periphery of the lungs in a healthy, normal patient. (u) B. Breath sounds in an asthmatic patient are usually obscured by wheezing. (u) C. Breath sounds are usually absent over an area of atelectasis. (u) D. Foreign body aspiration can present with stridor, wheezing or decreased breath sounds depending on where it has lodged.

Clinical Therapeutics/Pulmonology A patient is brought to the emergency room with acute onset of dyspnea and tachypnea. He has a long history of alcoholism and was involved in a motor vehicle accident two days ago. He is hypoxic with crackles auscultated bilaterally. Chest radiography reveals diffuse bilateral infiltrates which spare the costophrenic angle and air bronchograms, there was no cardiomegaly or pleural effusion noted. Oxygen saturation is 70%. Which of the following is the most important initial treatment? A. Tracheal intubation B. Bilateral chest tube insertion C. Type-specific packed cells D. Colloid solutions

(c) A. Tracheal intubation with lowest level of PEEP is required to maintain the PaO2 above 60mmHg or SaO2 above 90% in a patient with ARDS. (u) B. See A for explanation. (u) C. Fluids are the preferred treatment initially for hypovolemia. Type-specific packed cells are given when the patient's blood type is identified. Until then O negative packed cells are administered. (u) D. Use of crystalloid solutions are preferred to avoid pulmonary edema.

Diagnostic Studies/Pulmonology An adult patient who is HIV positive receives a PPD. He develops an area of induration that measures 8 mm after 48 hours. Which of the following is the most appropriate interpretation of this test result? Answers A. positive B. negative C. active infection D. falsely negative

Explanations (c) A. A reaction size of greater than or equal to 5 mm in a HIV positive patient is considered a positive tuberculin skin test reaction. (u) B. See A for explanation. (u) C. A positive PPD identifies patients that have been infected with Mycobacterium tuberculosis, but does not indicate whether the disease is currently active or inactive. (u) D. See A for explanation.

Scientific Concepts/Pulmonology Which of the following is the major pathogenetic mechanism that causes asthma? A. Airway inflammation B. Increased pulmonary secretions C. Presence of Ghon complexes D. Irreversible fibrosis

Explanations (c) A. Airway inflammation is the major pathogenetic mechanism that leads to the development of asthma. (u) B. Increased pulmonary secretions are the mechanism in chronic bronchitis. (u) C. The presence of Ghon complexes is noted in pulmonary tuberculosis. (u) D. Irreversible fibrosis of the lung parenchyma is associated with interstitial lung diseases.

Health Maintenance/Pulmonology A 17 year-old male who is trying out for the track team notes excessive coughing with chest tightness when running. Which of the following is the most appropriate preventive agent for this patient? Answers A. Albuterol inhaler (Proventil) B. Inhaled corticosteroids C. Aminophylline (Theo-Dur) D. Ipratropium (Atrovent)

Explanations (c) A. Albuterol is a beta-2 agonist that results in bronchodilation that makes this a useful agent in a patient with exercise-induced asthma when used just prior to exercise. (u) B. Inhaled corticosteroids are effective in exercise-induced asthma but are not acute acting and due to the side effect profile, they are not first line agents. (u) C. Aminophylline is not used as a first-line agent as a bronchodilator for patients with exercise-induced asthma. (u) D. Ipratropium main use is with suppression of mucous secretions and this is not a component of the exercise-induced asthma patient.

Diagnosis/Pulmonology A 36 year-old African American female comes to the clinic for an insurance physical which requires a chest x-ray. She denies any respiratory symptoms. Examination of her chest is negative. X-ray results show marked lymphadenopathy in the right paratracheal region. Angiotensin-converting enzyme (ACE) levels are elevated. Which of the following is the most likely diagnosis? A. Sarcoidosis B. Tuberculosis C. Pulmonary fibrosis D. Lymphoma

Explanations (c) A. Sarcoidosis is characterized by paratracheal lymphadenopathy and elevated ACE levels. It is more common in African American patients and may be asymptomatic. (u) B. See A for explanation. (u) C. See A for explanation. (u) D. See A for explanation.

History & Physical/Pulmonology A 24 year-old male presents in respiratory distress and appears quite ill. A Gram stain and culture of the sputum reveals gram-positive cocci in clumps and a chest x-ray reveals multiple patchy infiltrates with some cavitations. Which of the following is most likely to also be found in his medical history? A. IV drug abuse B. Alcohol abuse C. Poor dental hygiene D. HIV positive patient

Explanations (c) A. This patient has pneumonia caused by Staphylococcus aureus which is commonly associated with a history of intravenous drug use, influenza epidemics and the hospital setting. (u) B. A history of alcohol abuse is commonly seen with pneumonia caused by Klebsiella pneumoniae. (u) C. Poor dental hygiene is associated with pneumonia caused by anaerobes. (u) D. HIV positive patients are most at risk for development of pneumonia caused by Pneumocystis jiroveci.

History & Physical/Pulmonology A 30 year-old male presents with sudden onset of chills, fever, chest pain and a cough productive of greenish-brown sputum. On examination his temperature is 102 degrees F. He appears acutely ill and his respirations are shallow. Chest x-ray demonstrates left lower lobe consolidation. Which of the following findings would most likely be present on examination of his left lower lung? A. Hyperresonance B. Vesicular breath sounds C. Increased tactile fremitus D. Wheezing

Explanations (u) A. Hyperresonance is an abnormal sound as a result of air trapping as in COPD. (u) B. Vesicular breath sounds are the description of normal lung sounds. (c) C. Increased tactile fremitus occurs in the presence of fluid or a lung consolidation such as lobar pneumonia. (u) D. Wheezing is a musical noise sounding like a squeak caused by high velocity air flow through a narrow or obstructed airway.

Scientific Concepts/Pulmonology Which of the following conditions will produce a transudative pleural effusion? A. Kaposi's sarcoma B. Pneumonia C. Cirrhosis D. Mesothelioma

Explanations (u) A. Kaposi's sarcoma, pneumonia, or mesothelioma will produce a transudative pleural effusion. (u) B. See A for explanation. (c) C. Transudative pleural effusions result from alteration in the formation of pleural fluid, the absorption of pleural fluid, or both, by systemic factors. Local factors affecting pleural fluid absorption and/or formation produce exudative pleural effusions. (u) D. See A for explanation.

Health Maintenance/Pulmonology Which of the following is an indication for a pediatric patient to receive the 23-valent polysaccharide vaccine (Pneumovax)? A. Children at any age with a history of asthma B. All children at 2,4,6 and 12-18 months of age C. All children at 12-23 months of age in a two dose series D. Children age 24-59 months at high risk for invasive pneumococcal disease

Explanations (u) A. Pediatric patients with cystic fibrosis, not asthma, are included in the indications for vaccination with Pneumovax, however they must be at least 2 years old. (u) B. The 7-valent pneumococcal conjugate vaccine (Prevnar) is currently recommended to be given to children under the age of two on the schedule outlined. (u) C. While a two dose series is recommended for appropriate pediatric patients that receive Pneumovax, the recommended timing between doses is 3-5 years. (c) D. Pneumovax is licensed for use in children over the age of 23 months and is indicated for all pediatric patients at increased risk for pneumococcal disease.

History & Physical/Pulmonology Which of the following physical exam findings is consistent with moderate emphysema? A. Increased tactile fremitus B. Dullness to percussion C. Distant heart sounds D. Deviated trachea

Explanations (u) A. Physical examination findings in emphysema include a midline trachea, diffuse hyperresonant to percussion, and decreased tactile fremitus. (u) B. See A for explanation. (c) C. Distant heart sounds are common in emphysema patients due to hyperinflation of the lungs. (u) D. See A for explanation.

Clinical Therapeutics/Pulmonology A 17 year-old girl uses an albuterol inhaler to treat her asthma. She uses the inhaler as needed and reports symptoms occurring 3-4 days per week. She experiences symptoms at night no more than once a month. Her spirometry during her most recent office visit is normal. What is the appropriate medical management of this patient? A. Add an inhaled long-acting bronchodilator B. Add an inhaled steroid C. Add an inhaled long-acting bronchodilator and steroid D. No change to her medical regimen

Explanations (u) A. See B for explanation. (c) B. Symptoms occurring more than twice a week but less than daily meet severity criteria for mild persistent asthma. The appropriate next step in her therapy is to add an inhaled steroid. Adding an inhaled long-acting bronchodilator is only recommended for moderate persistent asthma and only after an inhaled steroid has been added. (u) C. See B for explanation. (u) D. See B for explanation.

Scientific Concepts/Pulmonology An 18 month-old male presents with his parents who report symptoms of a barking cough and intermittent stridor that has worsened over the past 12 hours. They note improvement in symptoms when he was taken outdoors to the cool night air. Which of the following is the most likely organism causing this patient's symptoms? A. Rubeola virus B. Adenovirus C. Influenza virus D. Para-influenza virus

Explanations (u) A. See D for explanation. (u) B. See D for explanation. (u) C. See D for explanation. (c) D. Croup is most often caused by parainfluenza virus.

Diagnostic Studies/Pulmonology An O2 saturation of 90% corresponds to what PO2 value? A. 90 mmHg B. 80 mmHg C. 70 mmHg D. 60 mmHg

Explanations (u) A. See D for explanation. (u) B. See D for explanation. (u) C. See D for explanation. (c) D. O2 sat values above 90% correspond with a PO2 >70 mmHg and values less than 94% represent hypoxemia. Less than 90% O2 sat warrants measurement of arterial blood gasses.

Diagnosis/Pulmonology A 53 year-old female status post abdominal hysterectomy 3 days ago suddenly develops pleuritic chest pain and dyspnea. On exam she is tachycardic and tachypneic with rales in the left lower lobe. A chest x-ray is unremarkable and an EKG reveals tachycardia. Which of the following is the most likely diagnosis? Answers A. atelectasis B. pneumothorax C. pulmonary embolism D. myocardial infarction

Explanations (u) A. Small atelectasis is commonly asymptomatic, while large atelectasis may produce signs of dyspnea and cough. Exam reveals absence of breath sounds in the area involved and dullness to percussion. A chest x-ray would reveal various findings dependent on the location of the atelectasis, but would not be normal. (u) B. While a pneumothorax commonly presents with pleuritic chest pain and dyspnea, exam would reveal the presence of diminished breath sounds and hyperresonance on the involved side. A chest x-ray would reveal presence of a pleural line on the expiratory chest x-ray. (c) C. Risk factors for pulmonary embolism include advanced age, surgery and prolonged bedrest. While the diagnosis of pulmonary embolism is difficult due to nonspecific clinical findings, the most common symptoms include pleuritic chest pain and dyspnea associated with tachypnea. Chest x-ray and EKG are usually normal. (u) D. While a myocardial infarction usually presents with dyspnea, the chest pain is not usually pleuritic in nature. An EKG would commonly reveal ST segment changes, which would be consistent with ischemia or infarct.

Clinical Therapeutics/Pulmonology A 22 month-old male infant presents with one day of barking cough preceded by three days of cold symptoms. On physical examination, his axillary temperature is 100.4°F and he has no stridor at rest. Inspiratory stridor is evident when he becomes agitated during the examination. There are no signs of respiratory distress or cyanosis. Which of the following is the most appropriate treatment for this patient? A. Nebulized albuterol B. Nebulized epinephrine C. Oral amoxicillin D. Oral dexamethasone

Explanations (u) A. Viral croup is an upper airway disease and there is no role for bronchodilator therapy. (u) B. Nebulized racemic epinephrine is only indicated in the treatment of croup in cases of moderate to severe rest stridor, respiratory distress, or hypoxia. (u) C. Croup is almost always a viral illness and antibiotics have no role in the treatment of this condition. (c) D. Corticosteroids are beneficial in the treatment of croup. Intramuscular administration has shown no benefit over oral administration.

Health Maintenance/Pulmonology A pediatric patient presents with a history of multiple recurrent respiratory infections associated with failure to thrive. A sweat chloride test is elevated. Which of the following is a common cause of death in patients with this condition? A. Diabetic ketoacidosis B. Pulmonary infection C. Intestinal obstruction D. Acute respiratory failure

Explanations (u) A. While patients with cystic fibrosis most likely will eventually develop insulin-dependent diabetes mellitus, diabetic ketoacidosis is not a common cause of death. (c) B. This patient has cystic fibrosis. The most common causes of death include pulmonary complications, such as infections, and terminal chronic respiratory failure associated with cor pulmonale. (u) C. While intestinal obstruction may occur in patients with cystic fibrosis, it is not a common cause of death. (u) D. See B for explanation.

Diagnosis/Pulmonology You are called to the nursery to see a male infant, born by uncomplicated vaginal delivery. He weighs 2,600 grams and has one deep crease on the anterior third of each foot. Respirations are 88 breaths/minute with expiratory grunting and intercostals retractions. He is cyanotic on room air and becomes pink when placed in 60% oxygen. Chest x-ray shows atelectasis with air bronchograms. Which of the following is the most likely diagnosis? Answers A. neonatal pneumonia B. congenital heart disease C. hyaline membrane disease D. chronic lung disease of prematurity

Explanations (u) A. While tachypnea, grunting, retractions and cyanosis may be signs of neonatal pneumonia, they are primarily late findings of progressive respiratory distress and would not be seen immediately at the time of delivery. A chest x-ray in pneumonia would also most commonly reveal an infiltrate or effusion. (u) B. While congenital heart disease may present with cyanosis, the chest x-ray will reveal a cardiac abnormality, such as cardiomegaly. (c) C. Hyaline membrane disease is the most common cause of respiratory distress in the premature infant. The infant typically presents with tachypnea, cyanosis and expiratory grunting. A chest x-ray reveals hypoexpansion and air bronchograms. (u) D. Chronic lung disease of prematurity is a complication in about 20% of infants with hyaline membrane disease. It is defined as respiratory symptoms, oxygen requirement and chest x-ray abnormalities at 1 month of age so it cannot be diagnosed at this time in this newborn.

Diagnosis/Pulmonology A 15 year-old male presents with a 1 week history of hacking non-productive cough, low grade fever, malaise and myalgias. Examination is unremarkable except for a few scattered rhonchi and rales upon auscultation of the chest. The chest x-ray reveals interstitial infiltrates and a cold agglutinin titer was negative. Which of the following is the most likely diagnosis? Answers A. acute bronchitis B. viral pneumonia C. mycoplasma pneumonia D. pneumococcal pneumonia

Explanations (u) A. While the patient's clinical symptoms of dry cough and rhonchi support this diagnosis, the chest x-ray would be normal or only show a mild increase in bronchovascular markings, not infiltrates. (c) B. The patient's clinical symptoms as well as chest x-ray findings and negative cold agglutinin titer are most consistent with viral pneumonia. (a) C. While the gradual onset of symptoms suggest mycoplasma, the negative cold agglutinin titer makes this less likely. (u) D. In older children the signs and symptoms of pneumococcal pneumonia are similar to an adult and consist of an abrupt onset of cough, fever and chills. The chest x-ray would reveal a lobar consolidation, not interstitial, picture.

Diagnosis/Pulmonology A 60 year-old patient returned from the recovery room to the floor following a subtotal gastrectomy. At 3 AM the next morning, the patient's temperature is 102° F (39° C) and pulse is 112/min. Which of the following is the most likely cause? A. wound infection B. atelectasis C. phlebitis D. shock

Explanations (u) A. Wound infection does not present this early. (c) B. Atelectasis is the most common pulmonary complication, affecting 25% of patients with abdominal surgery. It is more common in elderly and overweight patients and occurs within the first 12 to 24 hours postoperatively. (u) C. Phlebitis occurs more commonly after the second postoperative day. (u) D. In shock, the pulse is usually thready and the temperature is not elevated.

Clinical Intervention/Pulmonology A 32 year-old male with a history of Tetralogy of Fallot with poor right ventricular function presents for evaluation of sleep apnea. There is no evidence of deviated septum. Polysomnography reveals apneic episodes of 60 seconds in duration. Oxygen saturation falls to low levels. Which of the following is the first-line treatment in the management of this patient? A. Uvulopalatopharyngoplasty B. Nasal septoplasty C. Continuous positive airway pressure D. Antidepressants and oxygen

(u) A. Uvulopalatopharyngoplasty (UVVP) is a surgical procedure with resection of pharyngeal soft tissue and partial amputation of the soft palate and uvula. It is a procedure of last resort in most cases of sleep apnea. (u) B. Nasal septoplasty is performed if gross anatomic nasal septal deformity is present. (c) C. Continuous positive airway pressure prevents hypoxemia and maintains patency of the airway. (u) D. Antidepressants have no role in the treatment of sleep apnea.

Clinical Therapeutics/Pulmonology What is the mechanism of action of salmeterol (Serevent) in the treatment of asthma? A. Anti-inflammatory B. Immunotherapy for specific allergens C. Relaxing of bronchial smooth muscle D. Reduction of leukotriene production

Explanations (u) A. See C for explanation. (u) B. See C for explanation. (c) C. The mechanism of action for salmeterol is the relaxation of bronchial smooth muscle. (u) D. See C for explanation.

Diagnostic Studies/Pulmonology A 2 year-old presents with sudden onset of cough and stridor. On examination the child is afebrile and appears non- toxic with a respiratory rate of 42 breaths per minute. What is the next step in the evaluation of this patient? A. Lateral soft tissue x-ray of the neck B. Indirect laryngoscopy C. Finger sweep D. Chest x-ray

Explanations (u) A. See D for explanation. (u) B. See D for explanation. (u) C. See D for explanation. (c) D. Chest x-ray should be done first when foreign body aspiration is suspected.

Clinical Intervention/Pulmonology A 40 year-old male nonsmoker in good health undergoes a routine chest x-ray for an insurance physical. Results show an isolated, well-defined, coin lesion 1 cm in size. Which of the following is the next step in the evaluation of this problem? A. Review old radiographs B. Order chest CT C. Schedule lung biopsy D. Prepare for surgical lung resection

Explanations (c) A. The first and most important step in the radiographic evaluation is to review old radiographs to estimate doubling time, an important marker for malignancy. (u) B. See A for explanation. (u) C. See A for explanation. (u) D. See A for explanation.

Clinical Intervention/Pulmonology A 4 year-old boy is sent home from day care for a severe cough following one week of cold symptoms, including sneezing, conjunctivitis, and nocturnal cough. He presents with paroxysms of cough followed by a deep inspiration, and occasional post-tussive emesis. During severe paroxysms, he exhibits transient cyanosis. What is the most appropriate treatment for exposed contacts at his day care center? A. Macrolide prophylaxis B. Isolation C. Observation and treatment only if symptomatic D. Supportive care only

Explanations (c) A. All close contacts of a patient with pertussis should be treated with macrolide prophylaxis, regardless of age, immunization history, or symptoms. (u) B. Isolation of contacts is impractical and unnecessary. (u) C. Pertussis is rarely diagnosed before the paroxysmal stage, by which time exposure of contacts to the pathogen is assured. (u) D. While supportive care is essential in those contacts with symptoms, macrolide prophylaxis is mandatory in all contacts to prevent further spread of the illness.

History & Physical/Pulmonology A foreign body lodged in the trachea that is causing partial obstruction will most likely produce what physical examination finding? 19 Answers A. stridor B. aphonia C. inability to cough D. progressive cyanosis

Explanations (c) A. An inspiratory wheeze is called stridor, which indicates a partial obstruction of the trachea or larynx. (u) B. Aphonia, inability to cough and progressive cyanosis are seen with complete obstruction of the trachea, not partial obstruction. (u) C. See B for explanation. (u) D. See B for explanation.

History & Physical/Pulmonology A foreign body lodged in the trachea that is causing partial obstruction will most likely produce what physical examination finding? A. stridor B. aphonia C. inability to cough D. progressive cyanosis

Explanations (c) A. An inspiratory wheeze is called stridor, which indicates a partial obstruction of the trachea or larynx. (u) B. Aphonia, inability to cough and progressive cyanosis are seen with complete obstruction of the trachea, not partial obstruction. (u) C. See B for explanation. (u) D. See B for explanation.

Clinical Therapeutics/Pulmonology A 4 month-old infant is brought to the clinic by his mother with complaints of a cough for the past 3 weeks. Initially, symptoms included running nose, sneezing and an irritating cough. Over the past week the cough has changed to persistent staccato, paroxysmal forceful coughs ending with a loud inspiration. WBC is 20,0000/mcl with 72% lymphocytes. Which of the following is the drug of choice for managing this patient? A. Azithromycin (Zithromax) B. Ceftriaxone (Rocephin) C. Ampicillin (Unasyn) D. Gentamicin (Garamycin)

Explanations (c) A. Azithromycin terminates respiratory tract carriage of Bordetella pertussis. (u) B. Ceftriaxone does not eradicate Bordetella pertussis. (a) C. Ampicillin may be used for macrolide-intolerant patients however it is not the drug of choice. (u) D. Gentamicin does not eradicate Bordetella pertussis.

History & Physical/Pulmonology A 19 year-old male presents to the ED complaining of a sudden onset of dyspnea and left sided chest pain. He denies fever, chills, cough or sore throat. General survey shows that he is 6 feet 2 inches tall and weighs 135 lbs. Vital signs are BP 86/60 mmHg, HR 130 bpm, RR 28, temp. 98.6 degrees F. Which of the following would you likely find on examination of his thorax? A. Left-sided hyperresonance B. Increased tactile fremitus of left base C. Scattered rales throughout D. Increased anterior/posterior diameter

Explanations (c) A. Hyperresonance is present with accumulation of air in the pleural space. (u) B. Increased tactile fremitus is present with lung consolidation. (u) C. Mycoplasma pneumonia, common in young adults, presents with scattered rales audible throughout. (u) D. Increased A/P diameter is common in older patients with history of COPD.

History & Physical/Pulmonology A 55 year-old female presents to the emergency department with complaints of dyspnea, chest pain and coughing with hemoptysis. Past medical history includes breast cancer 5 years ago, currently in remission. Vital signs are Temp. 98.6 degrees F, BP 150/90 mmHg, P 110 bpm, RR 20. Physical examination shows her right leg swollen with pain on palpation of deep veins. Which of the patient's history or examination findings is most suggestive of a pulmonary embolus (PE)? A. Leg swelling and pain with palpation of deep veins B. Heart rate > 100 C. Hemoptysis D. Past history of cancer

Explanations (c) A. Leg swelling and pain with palpation of the deep veins are consistent with a DVT and increase the likelihood of a PE. (u) B. While a tachycardic rate may be present in a patient with a PE, it is not specific for a PE. (u) C. While hemoptysis may be present in a PE, it is not a specific finding. (u) D. History of cancer may place a patient at increased risk for PE, however is not the most suggestive finding in this patient.

Health Maintenance/Pulmonology What is the most effective prophylaxis against respiratory syncytial virus (RSV) infection in the general pediatric population? A. Proper hand-washing techniques B. A monoclonal antibody C. H. Influenzae B vaccine D. Oseltamivir (Tamiflu)

Explanations (c) A. Proper hand washing and reduction in exposure is most effective in general population to prevent RSV. (u) B. Prophylaxis with a monoclonal antibody has proven effective in high risk infants but is not indicated in the general pediatric population. (u) C. Prophylaxis with H. Influenzae B vaccine has reduced incidence of epiglottis, not RSV. (u) D. Oseltamivir is not indicated in the prophylaxis of RSV.

Diagnosis/Pulmonology An infant born at 30 weeks' gestation begins to have respiratory difficulty shortly after birth. Examination reveals rapid, shallow respirations at 80 per minute with associated intercostal retractions, nasal flaring and progressive cyanosis. Chest x-ray reveals the presence of air bronchograms and diffuse bilateral atelectasis. Which of the following is the most likely diagnosis? A. Respiratory distress syndrome B. Spontaneous pneumothorax C. Transient tachypnea syndrome D. Meconium aspiration syndrome

Explanations (c) A. Respiratory distress syndrome (hyaline membrane disease) is the most common cause of respiratory distress in a premature infant. This diagnosis is supported by the chest x-ray findings of air bronchograms and diffuse bilateral atelectasis, causing a ground-glass appearance. (u) B. Although spontaneous pneumothorax will present with respiratory distress at birth, the chest x-ray would reveal findings of lung collapse. (u) C. While transient tachypnea syndrome also may present at birth with respiratory distress, the chest x-ray would reveal findings of increased pulmonary vasculature markings, perihilar streaking and fluid in the interlobular fissures. (u) D. Meconium aspiration syndrome usually occurs in term or post-term infants. Typical chest x-ray findings include patchy infiltrates, coarse streaking of both lung fields, increased anteroposterior diameter and flattening of the diaphragm.

iagnosis/Pulmonology A 69 year-old male with a history of chronic lymphocytic leukemia presents to the clinic complaining of cough, dyspnea and production of copious amounts of foul smelling sputum. Physical examination reveals crackles at the lung bases. Chest x-ray shows dilated and thickened bronchi that appear as ring-like markings. Which of the following is the most likely diagnosis? A. Bronchiectasis B. Tuberculosis C. Adenocarcinoma D. Pulmonary fibrosis

Explanations (c) A. This patient has signs and symptoms consistent with bronchiectasis including CXR findings of dilated and thickened bronchi that may appear as tram-tracks or as ring-like markings. (u) B. TB would present with CXR findings in the apical or posterior segments of the upper lobes. (u) C. Radiographic findings of adenocarcinoma include enlarged nodule or mass; persistent opacity, atelectasis or pleural effusion. The sputum would not likely be foul smelling. (u) D. Pulmonary fibrosis does not present with dilated bronchi or ring-like markings on CXR

Clinical Therapeutics/Pulmonology A 70 year-old male is admitted to the ICU with fever, leukocytosis and purulent sputum. Sputum culture shows methicillin-resistant gram-positive cocci in clusters. Which of the following medications should be ordered for this patient? A. Vancomycin (Vancocin) B. Clindamycin (Cleocin) C. Azithromycin (Zithromax) D. Astreonam (Azactam)

Explanations (c) A. Vancomycin is the drug of choice for methicillin-resistant S. aureus. (u) B. Clindamycin is used if the patient is at risk for Legionella infection. (u) C. Azithromycin is more commonly used for H. influenza coverage. (u) D. Astreonam covers gram positive organisms but not methicillin-resistant S. aureus.

Diagnostic Studies/Pulmonology A 14 year-old male presents to the ED experiencing a severe asthma attack. His respiratory effort is shallow and he is using accessory muscles to breathe. Auscultation of his chest reveals no audible wheezing. Vital signs include BP 90/60 mmHg, P 160 bpm, RR 52. An arterial blood gas (ABG) is ordered. Normal ABG values at your institution are pH 7.35-7.45, CO2 35-45, pO2 80-95. Which of the following ABG findings suggests the poorest prognosis? A. pH = 7.27 pCO2 = 46 pO2 = 56 B. pH = 7.60, pCO2 = 18 pO2 = 80 C. pH = 7.44, pCO2 = 38 pO2 = 90 D. pH = 7.52, pCO2 = 28, pO2 = 80

Explanations (c) A. pH = 7.27 pCO2 = 46 pO2 = 56 is associated with the poorest prognosis in this patient. (u) B. See A for explanation. (u) C. See A for explanation. (u) D. See A for explanation.

Health Maintenance/Pulmonology An asymptomatic 60 year-old female with a 30 pack year history of smoking presents to the clinic requesting a chest x-ray to check for lung cancer. Which of the following do you recommend? A. A low-dose helical computed tomography (CT) B. Carcinoembryonic antigen (CEA) C. Serial chest radiographs to identify early stage malignancy D. No testing and referral for smoking cessation

Explanations (u) A. A low-dose helical computed tomography is very sensitive but is expensive, has increase false-positive tests and increases over-diagnosis. Mortality benefit remains to be proved. (u) B. CEA lacks clinical validation as a screening for lung cancer. (u) C. No major advisory organization recommends serial radiograph screening for lung cancer. (c) D. No major advisory organization recommends screening for lung cancer.

Clinical Therapeutics/Pulmonology A 6 year-old boy is brought to the pediatric clinic by his mother for an evaluation of his asthma. He coughs about 3 days out of the week with at least 2-3 nights of coughing. Which of the following would be the most appropriate treatment for this patient? A. Mast cell stabilizer B. Long acting beta agonist C. Leukotriene receptor antagonist D. Low dose inhaled corticosteroid

Explanations (u) A. A mast cell stabilizer is an alternative treatment but not the preferred treatment. (u) B. Long acting beta agonist can be used as adjunctive therapy with an anti-inflammatory. (u) C. Leukotriene receptor antagonists are an alternative treatment but not the preferred treatment. (c) D. Low dose inhaled corticosteroids are the preferred treatment for mild persistent asthma.

Clinical Therapeutics/Pulmonology An 85 year-old nursing home resident presents with abrupt onset of cough, sore throat, headache, myalgias, and malaise. On examination the patient's temperature is 102 degrees F; the rest of the exam is unremarkable. Nasal smear is positive for Influenza B. Which of the following is the treatment of choice in this patient? A. Amantadine (Symmetrel) B. Oseltamivir (Tamiflu) C. Acyclovir (Zovirax) D. Nevirapine (Viramune)

Explanations (u) A. Amantadine is only used to treat influenza A. (c) B. Oseltamivir is used to treat both influenza A and B. (u) C. Acyclovir is used to treat viral infections due to certain herpes viruses. (u) D. Nevirapine is used to treat infection due to HIV.

Diagnosis/Pulmonology A 40 year-old woman presents with 3 months of dry cough and intermittent low-grade fever. She is a non-smoker and has no significant family history or past medical history. A purified protein derivative (PPD) test was recently performed at work and was negative. On physical examination she is afebrile with stable vital signs. Lung auscultation reveals crackles in bilateral upper lobes. Chest x-ray shows hilar and mediastinal adenopathy, mild interstitial disease in the upper lung zones, and several small granulomas in both lungs. What is the most likely diagnosis? A. Asbestosis B. Cryptococcosis C. Sarcoidosis D. Tuberculosis

Explanations (u) A. Asbestosis typically presents as interstitial disease in the lower lungs and this patient has no known exposure to asbestos. (u) B. Cryptococcosis typically shows pleural-based nodules on x-ray and this patient has no known risk factors (HIV disease, COPD, chronic steroid use). (c) C. Sarcoidosis classically presents as a vague systemic illness with radiographic evidence of any or all of the following: granulomas, hilar and mediastinal adenopathy and interstitial infiltrate. (u) D. The patient's recent negative PPD makes tuberculosis unlikely.

Clinical Intervention/Pulmonology A 45 year-old male presents to the clinic complaining of morning sluggishness, daytime fatigue, headaches. He admits to drinking two cocktails each evening. His bed partner reports his loud cyclical snoring, breath cessation and thrashing movements of his extremities during sleep. BMI is 40. Heart examination reveals regular rate and rhythm without S3, S4, or murmur and lungs are clear to auscultation. Polysomnography shows apneic episodes lasting as long as 60 seconds. Which of the following clinical interventions would most likely provide for the acute cessation of apneic episodes? A. Avoidance of alcohol B. Weight loss C. Nasal continuous positive airway pressure D. Supplemental oxygen

Explanations (u) A. Avoidance of alcohol is also a necessary step in managing sleep apnea however does not provide immediate relief of apneic episodes. (u) B. Weight loss is also a necessary step in managing sleep apnea however does not provide immediate relief of apneic episodes. (c) C. Nasal CPAP is curative in many patients with sleep apnea. (u) D. Oxygen lessens the severity of nocturnal desaturation but may lengthen the apnea episodes.

Diagnosis/Pulmonology A 5 year-old male presents with a history of recurrent episodes of acute bronchitis, characterized by fever and productive cough. He has no known significant past medical history. His pulmonary examination reveals crackles in the bilateral lower lobes. The remainder of his physical examination is normal. Chest x-ray demonstrates platelike atelectasis and dilated, thickened airways in the middle and lower lungs. Which of the following is the most likely diagnosis? A. Acute bronchitis B. Bronchiectasis C. Pneumonia D. Tuberculosis

Explanations (u) A. Barring underlying pulmonary pathology,the chest x-ray in acute bronchitis should be normal. (c) B. Bronchiectasis typically presents as recurrent episodes of acute bronchitis. Platelike atelectasis and dilated and thickened airways, sometimes described as tram lines, are common radiographic findings. (u) C. While the history may suggest pneumonia, the radiographic findings do not support this diagnosis. (u) D. Tuberculosis would present with cavitating granuloma formation more commonly at the apices.

Health Maintenance/Pulmonology A mother of a newborn infant presents to the office concerned about reducing the risk of sudden infant death syndrome (SIDS). The infant was delivered at 39 weeks gestation weighing 7 pounds 9 ounces. There is no family history of SIDS and this is her first child. Which of the following is appropriate advice to reduce the risk of SIDS? A. Bottle feeding with soy formula B. Offer a pacifier at nap and bedtimes C. Have the infant sleep in the prone position D. Infant should sleep with the parents to allow close observation

Explanations (u) A. Bottle feeding with soy formula does not reduce the risk of SIDS. (c) B. Use of a pacifier during sleeping is a current recommendation to decrease the risk of SIDS. (u) C. Sleeping prone has been consistently shown to increase, not decrease, the risk of SIDS. (u) D. Bed sharing with parents has been shown to increase, not decrease, the risk of SIDS. This risk is increased in infants less than 4 months old and when older children are also present in the bed.

Diagnosis/Pulmonology A 74 year-old male with a history of coronary artery disease and atrial fibrillation presents to the clinic for follow-up of his shortness of breath. Patient's medications include amiodarone (Cordarone) and metoprolol (Lopressor). His chest x-ray reveals patchy ground-glass infiltrates. Which of the following is the most likely diagnosis? A. COPD B. Tuberculosis C. Bronchiectasis D. Pulmonary fibrosis

Explanations (u) A. COPD appears as hyperinflation with flattening of the diaphragm on chest radiograph. (u) B. Tuberculosis presents with pulmonary infiltrates on chest radiograph most often apical; cavitations may be seen with progressive primary tuberculosis. (u) C. Chest x-ray in bronchiectasis shows dilated and thickened bronchi that appear as ring-like markings. (c) D. Pulmonary fibrosis presents with ground-glass infiltrates on CXR and is often associated with certain medication use.

Clinical Therapeutics/Pulmonology A previously healthy 8 month-old boy is hospitalized for acute bronchiolitis. He has no known significant past medical or family history. On admission, he exhibits nasal flaring and retractions with a respiratory rate of 68, axillary temperature of 102.0 degrees F and O2 saturation of 86%. Which of the following medications is indicated? A. Prednisolone B. Oxygen C. Ceftriaxone (Rocephin) D. Palivizumab (Synagis)

Explanations (u) A. Corticosteroids are not indicated for the treatment of previously healthy infants with bronchiolitis. (c) B. Oxygen is an important supportive therapy for hypoxemic infants with bronchiolitis. Bronchodilators would also be initiated in this patient. (u) C. Antibiotics are not indicated in the treatment of bronchiolitis unless there is a secondary bacterial infection. (u) D. Palivizumab is used only for prevention of RSV infection.

Health Maintenance/Pulmonology A 33 year-old HIV-positive woman develops an 8mm area of induration following the administration of a purified protein derivative (PPD) test. Her chest radiograph shows no evidence of active tuberculosis (TB) infection. Which of the following is the most appropriate clinical intervention? A. Four-drug regimen for 4 months B. Isoniazid with Rifampin C. Observation only D. Repeat PPD and chest radiograph in 3 months

Explanations (u) A. Greater than 5 mm of induration is positive in an HIV-infected patient. A positive PPD and negative chest film is considered latent TB infection and, while requiring treatment, does not require the full four-drug regimen. (c) B. Isoniazid with Rifampin is recommended in HIV positive patients with a positive PPD and a negative chest x- ray. (u) C. Latent TB infection is associated with a risk of progression to tuberculosis and observation alone is inadequate. (u) D. Repeat screening is not helpful since the diagnosis of latent TB infection has already been established.

Clinical Intervention/Pulmonology A 67 year-old man presents complaining of gradually worsening fatigue and shortness of breath. He is a previous smoker with an 80 pack-year smoking history. He denies chest pain, night sweats, or hemoptysis. On physical examination, you note a very thin male who appears older than his stated age. Lung and heart sounds are barely audible to auscultation. Which of the following interventions is likely to alter the disease course? A. Inhaled bronchodilator therapy B. Inhaled steroid therapy C. Home oxygen D. Theophylline

Explanations (u) A. Inhaled bronchodilators afford symptomatic relief for some patients with COPD but do not alter the disease course. (u) B. Inhaled steroid therapy may reduce the number and severity of COPD exacerbations but has not been shown to alter the disease course. (c) C. Home oxygen therapy has been shown to prolong life in patients with COPD and alter the natural history of the disease. (u) D. Theophylline is a third-line agent for treating COPD and will not alter the natural history of the disease

Diagnostic Studies/Pulmonology Which of the following chest x-ray abnormalities would most likely be seen in a patient with hypersensitivity pneumonitis? A. Lobar consolidation B. Apical infiltration C. Granulomatous inflammation D. Diffuse nodular densities

Explanations (u) A. Lobar consolidation is seen in community-acquired pneumonia. (u) B. Apical infiltration is seen in tuberculosis. (u) C. Granulomatous inflammation is seen in sarcoidosis. (c) D. Diffuse nodular densities are seen in hypersensitivity pneumonitis.

Diagnosis/Pulmonology A 56 year-old female with a 35 pack year smoking history presents to the clinic with shortness of breath and cough. On examination, she is thin with no recent weight loss. She appears uncomfortable, breath sounds are diminished without adventitious sounds. Pulmonary function tests show a marked increase in total lung capacity (TLC) and a decreased FEV1. What is the most likely diagnosis for this patient? A. Persistent asthma B. Chronic obstructive pulmonary disease C. Idiopathic fibrosing interstitial pneumonia D. Sarcoidosis

Explanations (u) A. Lung function in asthma is evaluated by FEV1/FVC ratio with reduction noted with airflow obstruction. (c) B. Lung volume measurements in COPD reveal a marked increase in residual volume indicative of air trapping. (u) C. Pulmonary function testing in idiopathic fibrosing interstitial pneumonia shows a loss of lung volume with normal to increased airflow rates in interstitial lung disease. (u) D. Restrictive changes with decreased lung volumes and diffusing capacity are common in sarcoidosis.

Clinical Intervention/Pulmonology A 36 year-old male who is hospitalized because of severe injuries from a motor vehicle accident develops rapid onset of profound dyspnea. Initial chest x-ray shows a normal heart size with diffuse bilateral infiltrates. Follow-up chest x- ray shows confluent bilateral infiltrates that spare the costophrenic angles. Which of the following is the best clinical intervention for this patient? A. Provide supplemental oxygen B. Insert chest tube C. Tracheal intubation D. Administer corticosteroids

Explanations (u) A. Marked hypoxemia is refractory to treatment with supplemental oxygen in ARDS. (u) B. Chest tube insertion is not indicated in a patient with ARDS. (c) C. Treatment of hypoxemia in acute respiratory distress syndrome (ARDS) usually requires tracheal intubation. (u) D. Routine use of corticosteroids is not recommended in the management of ARDS.

Scientific Concepts/Pulmonology A 3 year-old male with cystic fibrosis develops pneumonia. Which of the following is the most likely etiology of the pneumonia? A. Escherichia coli B. Staphylococcus epidermidis C. Pseudomonas aueroginosa D. Streptococcus pneumoniae

Explanations (u) A. See C for explanation. (u) B. See C for explanation. (c) C. Initially in the first few months of life, respiratory infection is common with Staphylococcus aureus and Haemophilus influenzae, but after that Pseudomonas aueroginosa becomes the major causative organism for infections. (u) D. See C for explanation.

History & Physical/Pulmonology A 69 year-old male presents with complaint of increasing dyspnea over the past 6-8 months. The patient denies cough, chest pain or smoking history. Physical examination reveals inspiratory crackles at the bases and clubbing of the nails. Chest x-ray reveals interstitial fibrosis of the lower lungs, thickened pleura and calcified pleural plaques of the lateral chest wall. Pulmonary function testing shows a restrictive pattern with a decreased diffusing capacity. What information is most likely noted in this patient's history? A. Coal mining B. Silica exposure C. Textile work D. Asbestos exposure

Explanations (u) A. Patients with coal miners pneumoconiosis are typically asymptomatic with unremarkable pulmonary function tests. CXR shows small opacities in the upper lungs. (u) B. Patients with a history of silica exposure are also asymptomaticandhaveunaffectedpulmonaryfunction tests. CXR shows small rounded opacities throughout the lung and calcified hilar lymph nodes. (u) C. Textile workers present with an asthma-like disorder with chest tightness, cough and dyspnea that is worse on the first day back to work and improves as the week goes on. (c) D. Asbestos exposure often presents years later with increasing dyspnea and interstitial fibrosis of the lower lungs, thickened pleura and calcified pleura plaques. They will have a restrictive pattern on PFT.

Scientific Concepts/Pulmonology A 20 year-old male presents with 3 weeks of constitutional and upper respiratory symptoms, including malaise, sore throat, dry cough, and fever. Lung auscultation demonstrates diffuse crackles bilaterally. What is the most likely infectious agent involved? A. Respiratory syncytial virus B. Influenza virus C. Mycoplasma pneumoniae D. Streptococcus pneumoniae

Explanations (u) A. RSV is characterized by wheezing on auscultation and the course is typically 3-7 days. (u) B. Influenza pneumonia is characterized by a more precipitous onset and fulminant course. (c) C. The indolent course suggests an atypical pneumonia and Mycoplasma is the most common atypical agent. (u) D. Pneumococcal pneumonia is typically characterized by a more severe illness and more fulminant course.

Diagnostic Studies/Pulmonology What is the diagnostic modality of choice to diagnose cystic fibrosis (CF)? A. Chest radiograph B. Clinical features C. Sweat chloride concentration testing D. Genotyping

Explanations (u) A. Radiographic findings may suggest the diagnosis but are not specific. (u) B. While clinical features may suggest the need for testing they are not useful in confirming the diagnosis. (c) C. The standard for diagnosis is two positive sweat chloride concentration tests obtained on separate days or identification of CF mutations or an abnormal nasal potential difference measurement. (u) D. Genotyping screens for only a fraction of the known CF mutations.

Diagnostic Studies/Pulmonology A 56 year-old male with a 40 pack-year smoking history presents complaining of progressive shortness of breath. Spirometry reveals an FEV1 of 2 L (40% of predicted), an FVC of 4 L (80% of predicted) and an FEV1/FVC of 50%. These findings are most consistent with A. sarcoidosis. B. chronic bronchitis. C. interstitial lung disease. D. congestive heart failure.

Explanations (u) A. Sarcoidosis, interstitial lung disease and congestive heart failure most commonly produce a restrictive pattern on spirometry with a reduction in forced expiratory volume in one second (FEV1) and forced vital capacity (FVC) secondary to the decrease in total lung capacity (TLC), but the FEV1/FVC ratio is usually normal or increased, not decreased. (c) B. The reduced FEV1 and FEV1/FVC is characteristic of an obstructive pattern that is seen in chronic obstructive pulmonary diseases, such as chronic bronchitis. (u) C. See A for explanation. (u) D. See A for explanation.

Health Maintenance/Pulmonology You are seeing 62 year-old African American male for health maintenance. He is a former cigarette smoker with a 40 pack-year history. He quit smoking 10 years ago. He denies cough, hemoptysis, shortness of breath, chest pain, weight loss, or night sweats. What method of screening for lung cancer is appropriate in this patient? A. Chest radiograph B. Spiral CT of the chest C. Sputum cytology D. No screening is recommended

Explanations (u) A. Screening chest radiographs have not been shown to improve outcomes when used to screen asymptomatic patients. (u) B. While CT may yet prove valuable for screening, it is cost-prohibitive and has not yet been validated as a screening modality in asymptomatic patients. (u) C. Sputum cytology is not an effective screening tool. (c) D. No routine screening for lung cancer is recommended for asymptomatic smokers or former smokers.

Clinical Intervention/Pulmonology A 60 year-old female with a 30 pack year smoking history complains of new onset shortness of breath. On physical examination, dullness is noted on percussion with dimished breath sounds over her left base. Chest x-ray shows a new left pleural effusion. Which of the following is the next step in the management of this patient? A. Repeat chest x-ray in two months B. Perform diagnostic thoracentesis C. Order MRI of the chest D. Treat with antibiotic

Explanations (u) A. See B for explanation. (c) B. Diagnostic thoracentesis should be performed whenever there is a new pleural effusion and no clinically apparent cause. (u) C. See B for explanation. (u) D. See B for explanation.

Diagnostic Studies/Pulmonology You are evaluating a patient whom you suspect has asthma. You perform spirometry before and after administration of an inhaled short-acting bronchodilator. After administration of the bronchodilator, which of the following spirometry results would suggest reversibility? A. DecreaseinFEV1 B. IncreaseinFEV1 C. Decrease in FVC D. Increase in FVC

Explanations (u) A. See B for explanation. (c) B. In asthma, the airway obstruction should be at least partially relieved be a short-acting bronchodilator. This would be reflected in an increased forced expiratory volume in 1 second (FEV1). (u) C. The forced vital capacity (FVC) is not a function of obstruction and is generally normal in early mild asthma or lower than expected in severe or long-standing asthma. Either way, it is not expected to change with administration of a short-acting bronchodilator. (u) D. See C for explanation.

Diagnostic Studies/Pulmonology A 57 year-old man is being evaluated for shortness of breath. The following spirometric data are obtained: VC 4.90 L (predicted), 5.15 L (observed) 105% predicted FRC 3.99 L (predicted), 4.37 L (observed) 110% predicted RV 2.47 L (predicted), 3.17 L (observed) 128% predicted FEV1 3.50 L (predicted), 2.35 L (observed) 67% predicted These findings are consistent with which of the following? A. No demonstratable abnormality B. Restrictive lung disease C. Obstructive lung disease D. A ventilation/perfusion mismatch

Explanations (u) A. See C for explanation. (u) B. Restrictive lung disease would show decreased total lung capacity, vital capacity, and normal to increased FEV1. (c) C. Spirometry findings in obstructive lung disease typically show normal or increased total lung capacity, decreased vital capacity, prolonged FEV1, and increased residual volume. (u) D. A ventilation/perfusion scan would be abnormal with a pulmonary embolism.

Diagnosis/Pulmonology A 32 week preterm infant has an APGAR score of 9 at 5 minutes. Thirty minutes after delivery, tachypnea, retractions, and expiratory grunting are noted. Cyanosis and dyspnea appear with little response to oxygen. Physical examination reveals poor air movement bilaterally. A chest x-ray reveals air bronchograms and a fine reticular granular pattern. Which of the following conditions should be suspected? A. Atelectasis B. Diaphragmatic hernia C. Respiratory distress syndrome D. Pneumothorax

Explanations (u) A. Small areas of atelectasis usually are asymptomatic. While larger areas may present with similar clinical findings, the chest x-ray findings are not consistent with atelectasis. (u) B. Chest x-ray in a patient with a diaphragmatic hernia would not show a fine reticular granular pattern. (c) C. Clinical findings of increasing cyanosis unresponsive to oxygen therapy and the characteristic x-ray findings are most consistent with respiratory distress syndrome. (u) D. Chest x-ray in a patient with a pneumothorax would not show a fine reticular granular pattern.

Health Maintenance/Pulmonology A recent Haitian immigrant presents to the clinic for an employment physical examination before starting work at a local hospital. The patient has a history of receiving bacilli Calmette-Guerin (BCG) vaccination. Screening for tuberculosis for this employee should include which of the following tests? A. Sputum induction B. PPD skin test C. Chest x-ray D. No screening needed

Explanations (u) A. Sputum induction should not be used as a screening test for tuberculosis. (u) B. False-positive tuberculin skin test reactions can occur in persons previously vaccinated against M. tuberculosis with BCG. PPD should be avoided as a screening test in these patients. (c) C. Chest x-ray is the test of choice in patients where the PPD test is not indicated or in high-risk individuals. (u) D. This patient has recently emigrated from a possible endemic region and should be screened for tuberculosis.

Scientific Concepts/Pulmonology A 62 year-old male smoker presents to the clinic with the complaint of a chronic cough, hemoptysis, and weight loss. Chest CT shows a mass obstructing the bronchus with hilar and mediastinal lymph node abnormalities. Bronchoscopy with biopsy is performed. On reviewing pathology results you explain to the patient that his type of lung cancer is prone to early hematogenous spread, is rarely amenable to surgical resection and has a very aggressive course. What type of lung cancer is most likely in this patient? A. Squamous cell B. Small cell C. Large cell D. Adenocarcinoma

Explanations (u) A. Squamous cell carcinoma, large cell carcinoma, and adenocarcinoma spread more slowly and have the possibility of cure in early stages following resection and chemotherapy. (c) B. Small cell lung cancer is very aggressive with a median survival (untreated) of 6-18 weeks. -=-80-0 (u) C. See A for explanation. (u) D. See A for explanation.

Diagnostic Studies/Pulmonology Which of the following findings confirm the adequacy of a sputum specimen for Gram stain and culture? A. Decreased red blood cells B. Decreased bronchial epithelial cells C. Increased Gram positive cocci D. Increased polymorphonuclear leukocytes

Explanations (u) A. The presence or absence of red blood cells is not a criterion for determining the adequacy of a sputum sample. (u) B. The presence of bronchial epithelial cells confirms the sample came from the lower respiratory tract. (u) C. The presence of Gram positive cocci has no bearing on the adequacy of the sputum sample. (c) D. The presence of increased polymorphonuclear leukocytes and the absence of squamous epithelial cells are the criteria utilized to evaluate the adequacy of a sputum sample.

Diagnostic Studies/Pulmonology A 26 year-old man presents to the emergency room complaining of shortness of breath, palpitations, and tingling of the lips and fingers. He appears anxious and describes a sensation of impending doom. His ECG and plain chest radiograph are normal. Which of the following arterial blood gas findings would you expect in this patient? A. pH 7.32 pCO2 49 mm Hg bicarbonate 24 mEq/L B. pH 7.40 pCO2 40 mm Hg bicarbonate 25 mEq/L C. pH 7.50 pCO2 23 mm Hg bicarbonate 21 mEq/L D. pH 7.52 pCO2 40 mm Hg bicarbonate 44 mEq/L

Explanations (u) A. These values are consistent with a respiratory acidosis. (u) B. These values fall within the normal range. (c) C. This patient is suffering an acute anxiety attack and resultant hyperventilation and respiratory alkalosis. Labs show alkaline pH, depressed pCO2 and low to normal bicarbonate. (u) D. These values suggest metabolic alkalosis.

Clinical Intervention/Pulmonology A 22 year-old patient complains of sudden onset of chest pain accompanied by shortness of breath. The patient appears dyspneic. On examination, the trachea is deviated to the left, breath sounds are faint on the right, and the right chest is hyperresonant to percussion. The preferable treatment for this patient would be A. atracheostomy. B. insertion of a chest tube with underwater seal, left 2nd intercostal space. C. needle thoracotomy right 2nd intercostal space D. a lung scan for pulmonary embolus and begin heparin sodium (Heparin) therapy.

Explanations (u) A. Tracheostomy is indicated for upper airway obstruction, not spontaneous pneumothorax. (h) B. This patient has a tension pneumothorax on the right, and insertion of the chest tube on the left would be life threatening. (c) C. Decreased breath sounds and hyperresonance are noted on the side of the pneumothorax; tracheal deviation to the opposite side indicates development of a tension pneumothorax. Treatment consists of inserting a chest tube on the side of the pneumothorax and connecting to an underwater seal. (u) D. Pulmonary embolus may present with chest pain and dyspnea, but physical examination findings do not include decreased breath sounds with hyperresonance and tracheal deviation.

History & Physical/Pulmonology Which of the following is the most likely presentation of an acute pulmonary embolism (PE) in a patient without preexisting cardiac or pulmonary disease? A. Anginal chest pain B. Cough C. T achypnea D. Palpitations

Explanations (u) A. While it is quite common for PE to present with pleuritic chest pain, angina-like pain is only rarely reported. (u) B. Cough is reported in roughly one-third of patients with PE. But is not the most common presentation. (c) C. Tachypnea is the most common symptom in acute PE. (u) D. Palpitations are uncommonly reported in acute PE.

Clinical Intervention/Pulmonology Which of the following is an indication for hospitalization in a patient who has acute bronchiolitis? A. pulse oximetry of 94% on room air B. children between 4-6 months of age C. moderate tachypnea with feeding difficulties D. hyperinflation and interstitial infiltrates on chest x-ray

Explanations (u) A. A pulse oximetry reading of 94% on room air is equivalent to a PaO2 of approximately 80 mm Hg which indicates the child is not in severe respiratory distress. (u) B. Children less than 2 months of age require hospitalization. (c) C. Indications for hospitalization include moderate tachypnea with feeding difficulties. (u) D. Hyperinflation and interstitial infiltrates on chest x-ray are frequently seen with acute bronchiolitis and by themselves are not an indication for hospitalization.

Health Maintenance/Pulmonology A 3 year-old girl is diagnosed with atopic dermatitis. Which of the following disorders is this child at risk for in the future? A. Asthma B. Tinea pedis C. Squamous carcinoma D. Systemic lupus erythematosus (SLE)

Explanations (c) A. Up to 50% of patients with atopic dermatitis develop asthma and/or allergic rhinitis in the future. (u) B. Patients with atopic dermatitis are more likely to get superimposed viral or bacterial infections such as herpes simplex or staphylococcal, but they are not more at risk for fungal infections. (u) C. Patients with atopic dermatitis are at no greater risk for any skin cancer. (u) D. Lupus is a connective tissue disorder of the immune system, but unrelated to atopic dermatitis.

Clinical Intervention/Pulmonology A 59 year-old otherwise healthy female develops acute dyspnea and chest pain one week post total abdominal hysterectomy. Echocardiogram demonstrates normal heart size with normal right and left ventricular function. Lung scan demonstrates two segmental perfusion defects. Which of the following is the next step in the management of this patient? A. Anticoagulation B. Embolectomy C. Thrombolysis D. Inferior vena cava filter

Explanations (c) A. Anticoagulation is the treatment of choice in patients with pulmonary embolism with normal ventricular function and no absolute contraindications. (u) B. Embolectomy is not indicated as initial treatment of a pulmonary embolism in patients with normal ventricular function. (h) C. Thrombolysis is contraindicated in patients within 10 days of having major surgery. (u) D. An inferior vena cava filter is considered in patients with contraindications to anticoagulation therapy or failed anticoagulation therapy.

Diagnosis/Pulmonology A 4 year-old patient presents with episodic wheezing and a non-productive cough for the last 4 weeks. His symptoms are worse at night. Past medical history reveals a history of atopic dermatitis. Physical examination at this time is unremarkable. Which of the following is the most likely diagnosis? A. Asthma B. Bronchiolitis C. Croup D. Cystic fibrosis

Explanations (c) A. Asthma is a chronic inflammatory disorder of the airways. It is characterized by episodic or chronic symptoms of airflow obstruction, breathlessness, cough, wheezing, and chest tightness. The strongest identifiable predisposing factor for the development of asthma is atopy. (u) B. Bronchiolitis is common in infants and young children presenting with acute onset of cough, rhinorrhea, tachypnea, and expiratory wheezes. (u) C. Croup usually presents with a prodrome of upper respiratory tract symptoms followed by onset of a barking cough and stridor. (u) D. Cystic fibrosis is an autosomal recessive disease and is characterized by a chronic cough, sputum production, dyspnea, and wheezing. Steatorrhea, diarrhea, and abdominal pain are also common.

iagnostic Studies/Pulmonology Which of the following is the most common radiographic presentation of lung abscess? A. cavitation B. pleural thickening C. hilar mass D. hyperinflation

Explanations (c) A. Cavitation is seen with lung abscess or progressive primary tuberculosis. (u) B. Pleural thickening is noted in mesothelioma. (u) C. Hilar and mediastinal abnormalities are common on chest radiography in patients with lung cancer. (u) D. Hyperinflation is the main clinical feature in emphysema.

Clinical Intervention/Pulmonology Which of the following is a major contraindication to curative surgical resection of a lung tumor? Answers A. Liver metastases B. Vagus nerve involvement C. Non-malignant pleural effusion D. Chest wall invasion of the tumor

Explanations (c) A. Distant metastases, except for solitary brain and adrenal metastases are an absolute contraindication for pulmonary resection. Other absolute contraindications include MI within past 3 months, SVC syndrome due to metastatic tumor, bilateral endobronchial tumor, contralateral lymph node metastases and malignant pleural effusion. (u) B. See A for explanation. (h) C. See A for explanation. (u) D. See A for explanation.

Clinical Intervention/Pulmonology Which of the following is a major contraindication to curative surgical resection of a lung tumor? A. Liver metastases B. Vagus nerve involvement C. Non-malignant pleural effusion D. Chest wall invasion of the tumor

Explanations (c) A. Distant metastases, except for solitary brain and adrenal metastases are an absolute contraindication for pulmonary resection. Other absolute contraindications include MI within past 3 months, superior vena cava syndrome due to metastatic tumor, bilateral endobronchial tumor, contralateral lymph node metastases and malignant pleural effusion. (u) B. See A for explanation. (u) C. See A for explanation. (u) D. See A for explanation.

Diagnostic Studies/Pulmonology A patient should be tested for tuberculosis prior to being treated with A. etanercept (Enbrel). B. cyclosporine (Neoral). C. methotrexate (Rheumatrex). D. prednisone (Deltasone).

Explanations (c) A. Etanercept is an anti-cytokine agent used in the treatment of rheumatoid arthritis and has as a side effect the potential for serious infections. One of these side effects includes reactivation of dormant tuberculosis. (u) B. Cyclosporine, methotrexate, and prednisone do not have the requirement to check for tuberculosis prior to initiating treatment. (u) C. See B for explanation. (u) D. See B for explanation.

Clinical Intervention/Pulmonology A solitary pulmonary nodule is found on a pre-employment screening chest x-ray in a 34 year-old nonsmoking male. There are no old chest x-rays to compare. Which of the following is the most appropriate next step in the evaluation? A. CT scan of the chest B. Needle biopsy of the lesion C. Positron emission tomography of the chest D. Fiberoptic bronchoscopy

Explanations (c) A. In the absence of old x-rays in a nonsmoking individual less than 35 years old, CT scan of the chest is the next step in the evaluation of a solitary pulmonary nodule. (u) B. A needle biopsy would be indicated for a person greater than 35 years old and/or with a history of smoking to evaluate a solitary pulmonary nodule. (u) C. Positron emission tomography (PET scan) would be indicated if the CT scan was nonconclusive. (u) D. Fiberoptic bronchoscopy would be indicated only in the presence of a history of tobacco use or if the lesion was suggestive of malignancy.

Scientific Concepts/Pulmonology Which of the following types of pleural effusion result from increased production of fluid due to underlying inflammatory conditions? A. Exudate B. Empyema C. Transudate D. Hemothorax

Explanations (c) A. Increased production of fluid due to inflammatory or malignant processes results in an exudative pleural effusion. (u) B. Infection in the pleural space causes an empyema. (u) C. Transudates result from increased hydrostatic or decreased oncotic pressures across normal capillaries. (u) D. Bleeding into the pleural space causes a hemothorax.

History & Physical/Pulmonology Which of the following is the most likely to develop into a persistent cough in the adult patient? A. Pertussis B. Allergic rhinitis C. Pharyngitis D. Heart failure

Explanations (c) A. Pertussis is suspected in patients with persistent cough that lasts longer than 2-3 weeks. Allergic rhinitis, pharyngitis and heart failure are all potential causes of acute cough. (u) B. See A for explanation. (u) C. See A for explanation. (u) D. See A for explanation.

Scientific Concepts/Pulmonology Which of the following describes the pathophysiological changes of pulmonary sarcoidosis? A. granulomas and inflammation of alveoli, small bronchi and small blood vessels B. inflammation and destruction of the structural components of the bronchial wall C. increased permeability of the alveolar-capillary membrane and diffuse alveolar damage D. edema of the airways with eosinophils, neutrophils, and lymphocytes

Explanations (c) A. Sarcoidosis is characterized by granulomas and inflammation of alveoli, small bronchi and small blood vessels. (u) B. Bronchiectasis is characterized by inflammation and destruction of the structural components of the bronchial wall. (u) C. Acute respiratory distress syndrome is characterized by increased permeability of the alveolar-capillary membrane and diffuse alveolar damage. (u) D. Asthma is characterized by edema of the airways with eosinophils, neutrophils, and lymphocytes.

Clinical Intervention/Pulmonology A 60 year-old patient with COPD characteristic of emphysema presents with a cough and increased sputum production. The following information is noted: Temperature 100°F (37.8°C); Respiratory rate 20/min; Heart rate 88 beats/min; pH 7.44; PaO2 75 mmHg; PaCO2 40 mmHg; O2 saturation 92%. Physical examination is remarkable for increased AP diameter, diminished breath sounds without wheezes, rhonchi, or other signs of respiratory distress. Which of the following would be an appropriate treatment for this patient? A. Broad-spectrum antibiotic B. Admission to the hospital C. Oxygen at 6 L/min by nasal cannula D. Brief course of oral theophylline

Explanations (c) A. Sputum production is extremely variable from patient to patient, but any increase in sputum with a history of COPD reported by a patient must be regarded as potentially infectious and treated promptly. (u) B. Admission is only warranted if the patient's respiratory status requires ventilatory assistance. This patient's blood gases are unremarkable for a patient with COPD and the patient is not in respiratory distress. (u) C. Oxygen therapy should only be used for severe hypoxemia and should only be given at a low concentration, such as 2 L/min. Higher dose oxygen may stop the hypoxemic ventilatory drive. (u) D. Oral theophylline is considered a secondary bronchodilator. The use of a metered-dose inhaler would be a preferable first-line treatment if this method of treatment were chosen.

Clinical Therapeutics/Pulmonology A 47 year-old HIV positive female presents with a complaint of a nonproductive cough. She is febrile, tachypneic and tachycardic. Lung exam reveals bilateral rales. Chest x-ray shows diffuse interstitial infiltrates. What is the recommended treatment in this patient? A. trimethoprim-Sulfamethoxazole (Bactrim) B. tetracycline (Sumycin) C. amantadine (Symmetrel) D. ticarcillin (Ticar)

Explanations (c) A. TMP-SMX is the drug of choice for all forms of pneumocystis. (u) B. Tetracycline, amantadine and ticarcillin are not effective against PCP. (u) C. See B for explanation. (u) D. See B for explanation.

Clinical Therapeutics/Pulmonology A patient presents with occasional wheezing and chest tightness that occurs approximately once a week and at night only about once a month. Peak expiratory flow is 85% of predicted. Which of the following is the most appropriate initial treatment? A. Albuterol (Proventil) inhaler B. Montelukast (Singular) C. Salmeterol (Serevent) inhaler D. Sustained release theophylline

Explanations (c) A. This patient has mild intermittent asthma which is initially treated with inhaled beta 2-agonists as needed. No long-term control medications are indicated. (u) B. Leukotriene modifiers, such as montelukast, may be added to the treatment of uncontrolled asthma as a long- term controller after the initiation of inhaled corticosteroids. (u) C. Long-acting beta 2-agonists, such as salmeterol, are indicated for long-term control of asthma that is categorized as moderate persistent to severe persistent. (u) D. Sustained release theophylline is an alternative treatment for asthma that is at least categorized as mild persistent; however its narrow therapeutic window and side effects limit its use.

History & Physical/Pulmonology A 45 year-old male presents with sudden onset of pleuritic chest pain, productive cough and fever for 1 day. He relates having symptoms of a "cold" for the past week that suddenly became worse yesterday. Which of the following findings will most likely be seen on physical examination of this patient? Answers A. spoken "ee" heard as "ay" B. hyperresonant percussion note C. wheezes over the involved area D. vesicular breath sounds over involved area

Explanations (c) A. This patient most likely has a bacterial pneumonia with consolidation, which would produce egophony, where a spoken "ee" is heard as "ay." (u) B. Consolidation from bacterial pneumonia causes findings of dullness to percussion, late inspiratory crackles and bronchial breath sounds over the involved area. (u) C. See explanation B. (u) D. See explanation B.

Diagnosis/Pulmonology A 24 year-old male presents complaining of a 9 month history of increasing shortness of breath, dyspnea on exertion, and a cough productive of white sputum, mostly in the mornings. He denies orthopnea, PND, peripheral edema, fever, chills, night sweats, recent changes in weight, palpitations, chest pain, food intolerances, or other complaints. Patient has a history of recurrent lung infections. He states that his father had chronic pulmonary problems and died at age 42 from unknown lung disease. The patient denies smoking, alcohol or illicit drug use. On physical examination, the respiratory rate is 22 per minute, pulse of 98 bpm, temperature of 98.7 degrees. Pulmonary exam reveals end-expiratory wheezes bilaterally and hyperresonance to percussion. His cardiac exam is normal. Chest x- ray shows decreased lung markings. ECG is normal. Pulmonary function tests show an FEV1 63% of expected and residual capacity is 123% of expected. Which of the following is the most likely diagnosis? A. Emphysema B. Pulmonary fibrosis C. Ventricular septal defect D. Congestive heart failure

Explanations (c) A. This person has an obstructive lung disease based on PFTs. Emphysema is the most likely diagnosis, and may be related to alpha-1 antitrypsin deficiency based on family history and lack of smoking history and young age. (u) B. The PFTs from a person with pulmonary fibrosis would be consistent with a restrictive pattern. This patient has an obstructive pattern of lung disease. (u) C. Ventricular septal defect will have a systolic murmur associated with it. (u) D. Congestive heart failure might explain some of the symptoms of this patient (increasing shortness of breath and DOE), he denies other common symptoms, such as orthopnea and peripheral edema. CHF should not result in changes in the PFTs.

Clinical Intervention/Pulmonology A 21 year-old male presents to the ED with increasing dyspnea and pleuritic chest pain of sudden onset after getting hit in the left side of the chest during a bar fight. Examination reveals moderate respiratory distress with absence of breath sounds and hyperresonance to percussion on the left, with tracheal deviation to the right. Which of the following is the most appropriate next step? Answers A. order a V/Q scan B. order a chest x-ray C. administer a sclerosing agent D. insert large bore needle into left 2nd ICS stat

Explanations (h) A. A V/Q scan is indicated in suspected cases of pulmonary embolism. (u) B. Patients in respiratory distress and evidence of a tension pneumothorax, such as tracheal deviation, should have treatment initiated without waiting on a chest x-ray to be taken. (h) C. Pleurodesis by administration of a sclerosing agent is indicated in treatment of recurrent, not traumatic, pneumothorax. (c) D. Simple aspiration by insertion of a needle into the involved side will decompress the tension pneumothorax until a chest tube can be inserted.

Clinical Intervention/Pulmonology A patient with severe COPD presents to the Emergency Department with a 3 day history of increasing shortness of 50 breath with exertion and cough productive of purulent sputum. An arterial blood gas reveals a pH of 7.25, PaCO2 of 70 mmHg and PaO2 of 50 mmHg. He is started on albuterol nebulizer, nasal oxygen at 2 liters per minute, and an IV is started. After one hour of treatment, his arterial blood gas now reveals a pH of 7.15, PaCO2 100 mmHg and PaO2 of 70 mmHg. Which of the following is the most appropriate next step in his treatment? A. Decrease the oxygen flow rate. B. Administer oral corticosteroids. C. Intubate the patient. D. Administer salmeterol (Serevent)

Explanations (h) A. Decreasing the oxygen flow rate would be harmful as it would decrease the amount of oxygen delivered to the patient. (u) B. Administration of steroids is an important treatment modality but this patient is in respiratory failure and needs more immediate therapy. (c) C. This person has increasing respiratory failure as indicated by the raising PaCO2 levels. Intubation is required at this time. (h) D. Long-acting beta agonist therapy such as salmeterol is not utilized for rescue therapy.

Diagnostic Studies/Pulmonology A 23 year-old female with history of asthma for the past 5 years presents with complaints of increasing shortness of breath for 2 days. Her asthma has been well controlled until 2 days ago and since yesterday she has been using her albuterol inhaler every 4-6 hours. She is normally very active, however yesterday she did not complete her 30 minutes exercise routine due to increasing dyspnea. She denies any cough, fever, recent surgeries or use of oral contraceptives. On examination, you note the presence of prolonged expiration and diffuse wheezing. The remainder of the exam is unremarkable. Which of the following is the most appropriate initial diagnostic evaluation prior to initiation of treatment? Answers A. chest x-ray B. sputum gram stain C. peak flow D. ventilation - perfusion scan

Explanations (u) A. A chest x-ray should be ordered in an asthmatic patient only if you are concerned about the presence of pneumonia or pneumothorax, neither of which is supported by the H&P findings noted above. (u) B. A sputum gram stain is performed in patients who you suspect have an infectious process, such as pneumonia. (c) C. A peak flow reading will help you to gauge her current extent of airflow obstruction and is helpful in monitoring the effectiveness of any treatment interventions. (u) D. A ventilation-perfusion scan (V/Q scan) is indicated in cases of suspected pulmonary embolism. The patient above does not have any risk factors that would lead you to suspect such a diagnosis.

Diagnostic Studies/Pulmonology A 23 year-old female with history of asthma for the past 5 years presents with complaints of increasing shortness of breath for 2 days. Her asthma has been well-controlled until 2 days ago. Since yesterday, she has been using her albuterol inhaler every 4 to 6 hours. She is normally very active, however yesterday she did not complete her 30 minute exercise routine due to increasing dyspnea. She denies any cough, fever, recent surgeries, or use of oral contraceptives. On examination, you note the presence of prolonged expiration and diffuse wheezing. The remainder of the exam is unremarkable. Which of the following is the most appropriate initial diagnostic evaluation prior to initiation of treatment? A. Chest x-ray B. Sputum gram stain C. Peak flow D. Ventilation-perfusion scan

Explanations (u) A. A chest x-ray should be ordered in an asthmatic patient only if you are concerned about the presence of pneumonia or pneumothorax, neither of which is supported by the H&P findings noted above. (u) B. A sputum gram stain is performed in patients who you suspect have an infectious process, such as pneumonia. (c) C. A peak flow reading will help you to gauge her current extent of airflow obstruction and is helpful in monitoring the effectiveness of any treatment interventions. (u) D. A ventilation-perfusion scan (V/Q scan) is indicated in cases of suspected pulmonary embolism. The patient above does not have any risk factors that would lead you to suspect such a diagnosis.

Diagnosis/Pulmonology A 28 year-old man presents to the emergency department complaining of sudden onset of shortness of breath associated with sharp right-sided chest pain increased with breathing. On physical examination, respirations are 20 per minute and blood pressure is 120/76 mm Hg. Auscultation of the chest reveals absent breath sounds over the right apex with normal heart sounds. Percussion of the right apex is noted to be hyperresonant. Which of the following is the most likely diagnosis? A. Hemothorax B. Pneumothorax C. Pulmonary embolus D. Foreign body aspiration

Explanations (u) A. A hemothorax usually results from trauma. While breath sounds would be absent over the involved area, the percussion note would be dull, not hyperresonant. (c) B. This patient most likely has a spontaneous pneumothorax which is supported by the presenting symptoms of sudden onset of dyspnea and pleuritic chest pain as well as the physical exam findings of absent breath sounds and hyperresonance to percussion. (u) C. While a pulmonary embolism most commonly presents with dyspnea and pain on inspiration, the physical examination is often unremarkable and would not reveal the findings of absent breath sounds and hyperresonance to percussion. (u) D. Foreign body aspiration is most common in children and the elderly. Aspiration of a food bolus is the most common cause in adults which leads to a history of a choking episode followed by dysphagia, wheezing and coughing. Physical examination findings are dependent on the location of the obstruction.

Clinical Intervention/Pulmonology A 42 year-old male is brought to the emergency department with a stab wound to his right lateral chest wall. On physical examination, the patient is stable with decreased breath sounds on the right with dullness to percussion. An upright chest x-ray reveals the presence of a moderate pleural effusion. Subsequent diagnostic thoracentesis contains bloody aspirate. Which of the following is the next most appropriate intervention? A. Thoracotomy B. Needle aspiration C. Close observation D. Tube thoracostomy

Explanations (u) A. A thoracotomy is indicated in a small percentage of patients that do not respond to IV administration of fluids and evacuation of the hemothorax via tube thoracostomy. (u) B. Needle aspiration as treatment for a hemothorax is not recommended as it fails to adequately drain the bloody fluid. (u) C. Close observation is only indicated for small effusions in a patient that remains hemodynamically stable. (c) D. This patient has a hemothorax. Drainage of a hemothorax is best obtained through insertion of a chest tube (tube thoracostomy).

Clinical Therapeutics/Pulmonology Which of the following classes of medications is most likely to cause a persistent cough? A. Tricyclic antidepressants B. Calcium channel blockers C. Beta-adrenoceptor blocking agents D. Angiotensin converting enzyme inhibitors

Explanations (u) A. Adverse effects of tricyclic antidepressants are due to blockage of acetylcholine receptors causing such symptoms as blurred vision, dry mouth, urinary retention and constipation. (u) B. Side effects are uncommon with calcium channel blockers, but include constipation, dizziness, headache and a feeling of fatigue. (u) C. Common adverse effects of beta-adrenoceptor blocking agents include bradycardia and central nervous system symptoms, such as fatigue or insomnia. (c) D. Common side effects of angiotensin converting enzyme inhibitors (ACE inhibitors) include a dry cough.

Clinical Intervention/Pulmonology A 3 month-old male presents with a hoarse cough and thick purulent rhinorrhea for the past 2 days. The mother noted that yesterday he appeared to get worse and seemed to have increasing problems breathing and trouble feeding. Examination reveals a temperature of 100.2 degrees F and respiratory rate of 80/minute with nasal flaring and retractions. Lung examination reveals a prolonged expiratory phase with inspiratory rales. He is tachycardic. Pulse oximetry reveals oxygen saturation of 89%. Chest x-ray reveals hyperinflation with diffuse interstitial infiltrates. Which of the following is the most appropriate intervention? Answers A. antibiotics B. hospitalization C. inhaled corticosteroids D. Racemic epinephrine

Explanations (u) A. Antibiotics are utilized to treat bacterial, not viral, illnesses. (c) B. This infant most likely has bronchiolitis. While most cases are mild and can be treated at home, hospitalization is recommended for infants with hypoxia on room air, moderate tachypnea with feeding difficulties and marked respiratory distress with retractions. Additionally hospitalization is recommended for infants less than 2-3 months of age, a history of apnea or an underlying chronic cardiopulmonary disease. (u) C. The use of corticosteroids in children with bronchiolitis has not been studied and does not appear to be helpful. (u) D. Racemic epinephrine is not indicated in the treatment of bronchiolitis.

Clinical Intervention/Pulmonology A 3 month-old male presents with two days of worsening hoarse cough and thick purulent rhinorrhea associated with increasing problems breathing and trouble feeding. Examination reveals a temperature of 100.2 degrees F and respiratory rate of 80/minute with nasal flaring and retractions. Lung examination reveals a prolonged expiratory phase with inspiratory rales. He is tachycardic. Pulse oximetry reveals oxygen saturation of 89%. Chest x-ray reveals hyperinflation with diffuse interstitial infiltrates. Which of the following is the most appropriate intervention? A. Antibiotics B. Hospitalization C. Inhaled corticosteroids D. Racemic epinephrine

Explanations (u) A. Antibiotics are utilized to treat bacterial, not viral, illnesses. (c) B. This infant most likely has bronchiolitis. While most cases are mild and can be treated at home, hospitalization is recommended for infants with hypoxia on room air, moderate tachypnea with feeding difficulties and marked respiratory distress with retractions. Additionally hospitalization is recommended for infants less than 2-3 months of age, a history of apnea or an underlying chronic cardiopulmonary disease. (u) C. The use of corticosteroids in children with bronchiolitis has not been studied and does not appear to be helpful. (u) D. Racemic epinephrine is not indicated in the treatment of bronchiolitis.

History & Physical/Pulmonology A patient presents with respiratory complaints. Chest x-ray reveals calcification of the hilar nodes with an eggshell pattern. Which of the following occupations is most consistent with these chest x-ray findings? A. building demolitioners B. coal miners C. sandblasters D. farmers

Explanations (u) A. Asbestosis is the most likely occupational risk and has an indistinct heart border appearance on CXR described as looking like "ground glass". (u) B. The CXR of a coal miner consists of irregular opacities ranging from a reticular pattern to a nodular pattern. (c) C. Silicosis can occur in sandblasters and produce a CXR appearance of calcification of the hilar nodes with an "eggshell" pattern with long term exposure (u) D. Farmers lung results from spores and produces a hypersensitivity pneumonitis. The CXR would show a patchy fibrosis.

History & Physical/Pulmonology On physical examination you note diminished breath sounds over the right lower lobe with decreased tactile fremitus and dullness to percussion. Which of the following is the most likely cause? Answers A. asthma B. consolidation C. pneumothorax D. pleural effusion

Explanations (u) A. Asthma is characterized by decreased tactile fremitus, but would have resonant to hyperresonant percussion, not dullness. (u) B. Consolidation from pneumonia is characterized by dullness to percussion, but would have an increased, not decreased, tactile fremitus. (u) C. A pneumothorax is characterized by decreased to absent tactile fremitus, but would have a hyperresonant percussion note, not dullness. (c) D. A decreased tactile fremitus and dullness to percussion would be found in a pleural effusion.

Diagnostic Studies/Pulmonology A 42 year-old male smoker presents for further evaluation of a 4 cm solitary pulmonary nodule discovered on a recent chest x-ray. Which of the following diagnostic tests is indicated next? A. Bone scan B. Thoractotomy C. Mediastinoscopy D. CT scan of chest

Explanations (u) A. Bone scanning is indicated for evaluation of bone metastases, most commonly secondary to cancer of the breast or prostate. (u) B. Diagnostic thoractotomy is indicated for biopsy of the lesion should the CT scan of the chest indicate a suspicious malignant lesion that is inaccessible to thoracoscopy. (u) C. Mediastinoscopy can be utilized to further evaluate any enlarged mediastinal lymph nodes that may be found on the CT scan of the chest, but is not indicated prior to the CT scan. (c) D. A CT scan of the chest is needed to further evaluate the characteristics of the solitary pulmonary nodule and to determine lymph node involvement or presence of multiple lesions.

Diagnosis/Pulmonology A patient who appears very anxious enters the office complaining of dizziness with perioral and extremity paresthesias. She vaguely describes some chest discomfort. Physical examination is unremarkable, except for moderate tachypnea with obvious sighing respiration. This clinical picture is most consistent with A. bronchial asthma. B. hyperventilation syndrome. C. spontaneous pneumothorax. D. emphysema.

Explanations (u) A. Bronchial asthma attacks are associated with increased dyspnea and prolonged expiration. Patients may use accessory muscles of respiration as part of this acute condition. (c) B. Anxiety may result in hyperventilation that can result in perioral numbness and paresthesias of the extremities. These paresthesias are due to decreased CO2 in the blood stream that results from the hyperventilation. Anxious patients also will have nondescript chest pain as part of this condition and may also complain of dizziness. (u) C. Spontaneous pneumothorax patients will primarily complain of significant chest pain along with their dyspnea. These patients will not have perioral or extremity paresthesias. (u) D. Emphysema alone will not result in hyperventilation or the production of perioral or extremity paresthesias and is a chronic progressive rather than an acute onset condition.

Health Maintenance/Pulmonology Which of the following is the most effective way for patients with persistent asthma to monitor the severity of their symptoms? A. call the health care provider regularly B. keep a diary of symptoms C. monitor peak flow D. ask a family member to monitor symptoms

Explanations (u) A. Calling the health care provider regularly or asking a family member to monitor symptoms is not effective in patients understanding how to manage their asthma symptoms. (u) B. Keeping a diary may be effective for patients to understand their cause of symptoms, but would not be effective in helping to manage their symptoms. (c) C. Monitoring peak flow is the most effective way for the patients and health care providers to manage symptoms and guide treatment. (u) D. See A for explanation.

Clinical Therapeutics/Pulmonology Topic: 1e Author: Donna Yeisley A 2 month-old infant has been diagnosed with pneumonia due to Chlamydia trachomatis. Which of the following is the treatment of choice? A. Ceftriaxone (Rocephin) B. Doxycycline C. Levofloxacin (Levaquin) D. Erythromycin

Explanations (u) A. Ceftriaxone is a third-generation cephalosporin that may be safely used in children, however is not indicated for the treatment of Chlamydial pneumonia. (h) B. Doxycycline is a tetracycline and is contraindicated in children under eight years of age secondary to damaging effects on bone and teeth enamel. (h) C. Levofloxacin is a fluoroquinolone and is contraindicated for use in children under 18 years of age secondary to damaging effects that may occur with growing cartilage. (c) D. Erythromycin or sulfisoxazole is the treatment of choice for an infant with Chlamydial pneumonia.

Diagnostic Studies/Pulmonology A patient presents with a history of progressive worsening of dyspnea over the past several years. He gives a history of having worked as a ship builder for over 50 years. He denies any alcohol or tobacco use. On examination you note clubbing and inspiratory crackles. Which of the following chest x-ray findings support your suspected diagnosis? Answers A. hyperinflation and flat diaphragms B. interstitial fibrosis and pleural thickening C. cavitary lesions involving the upper lobes D. "eggshell" calcification of hilar lymph nodes

Explanations (u) A. Chest x-ray findings of hyperinflation and flat diaphragms suggest long-standing chronic obstructive lung disease. (c) B. This patient most likely has asbestosis, which is supported by his occupation as a ship builder and clinical presentation as noted above. Chest x-ray findings include interstitial fibrosis, pleural thickening and calcified pleural plaques on the diaphragm or lateral chest wall. (u) C. Chest x-ray findings of cavitary lesions involving the upper lobes suggest pulmonary tuberculosis. (u) D. Chest x-ray findings of "eggshell" calcification of hilar lymph nodes strongly supports the diagnosis of silicosis.

Diagnostic Studies/Pulmonology A 62 year-old homeless patient presents complaining of fever, weight loss, anorexia, night sweats and a chronic cough that recently became productive of purulent sputum that is blood streaked. On physical examination, the patient appears chronically ill and malnourished. Which of the following chest x-ray findings supports your suspected diagnosis? A. Hyperinflation and flat diaphragms B. Interstitial fibrosis and pleural thickening C. Cavitary lesions involving the upper lobes D. "Eggshell" calcification of hilar lymph nodes

Explanations (u) A. Chest x-ray findings of hyperinflation and flat diaphragms suggest long-standing chronic obstructive lung disease. (u) B. Interstitial fibrosis and pleural thickening on a chest x-ray are found in cases of interstitial lung disease. (c) C. This patient most likely has tuberculosis. A chest x-ray finding of cavitary lesions involving the upper lobes would support this suspected diagnosis. (u) D. Chest x-ray findings of "eggshell" calcification of hilar lymph nodes strongly supports a diagnosis of silicosis.

Clinical Therapeutics/Pulmonology Which of the following is the greatest risk factor for the development of drug resistance in the treatment of tuberculosis? Answers A. A child with cystic fibrosis B. An elderly patient in a nursing home C. Non-adherence to prescribed drug regimen D. Patients with a history of diabetes mellitus

Explanations (u) A. Children with cystic fibrosis are at an increased risk for various lung infections, but not drug resistance. (u) B. While institutionalized patients, such as nursing home residents, are at increased risk for infection with Mycobacterium tuberculosis, the patient is not at increased risk for drug resistance. (c) C. Non-adherence to prescribed drug regimens is a risk factor for the development of drug resistance as well as being a major cause of treatment failure and continued transmission of tuberculosis. Patient education about the importance of drug therapy is important in an attempt to avoid this. (u) D. Patients with a history of diabetes mellitus are at increased risk for active disease, not drug resistance.

Health Maintenance/Pulmonology A 62 year-old female is admitted to a nursing home during an outbreak of influenza. In review of her records, you note that she did not receive the flu vaccine this year. Which of the following is the most appropriate drug of choice for influenza prophylaxis in this patient? Answers A. Ciprofloxin (Cipro) B. Zanamivir (Relenza) C. Clarithromycin (Biaxin) D. Alpha-2b interferon (Avonex)

Explanations (u) A. Ciprofloxin is indicated for postexposure prophylaxis of anthrax. (c) B. Either zanamivir or oseltamivir are indicated for prophylactic use against influenza A or B. (u) C. Clarithromycin is indicated for prophylaxis against disseminated Mycobacterium avium complex. (u) D. Alpha-2b interferon is indicated for treatment of several disorders, such as chronic hepatitis B & C, but has no role in prophylactic treatment of any condition.

Diagnostic Studies/Pulmonology Which of the following is essential to make a diagnosis of cystic fibrosis? Answers A. Positive family history B. Elevated sweat chloride C. Recurrent respiratory infections D. Elevated trypsinogen levels

Explanations (u) A. Cystic fibrosis is a genetic disease, but a positive family history in and of itself is not enough to diagnose the condition. (c) B. The diagnosis of cystic fibrosis is made only after an elevated sweat chloride test or demonstration of a genotype consistent with cystic fibrosis. (u) C. While recurrent respiratory infections are a classic presentation of cystic fibrosis, the diagnosis relies on confirmation, as noted in explanation B. (u) D. Trypsinogen levels are used as a neonatal screening test and if elevated should be followed by more definitive testing to confirm the diagnosis.

Diagnostic Studies/Pulmonology Which of the following is essential to make a diagnosis of cystic fibrosis? A. Positive family history B. Elevated sweat chloride C. Recurrent respiratory infections D. Elevated trypsinogen levels

Explanations (u) A. Cystic fibrosis is a genetic disease, but a positive family history in and of itself is not enough to diagnose the condition. (c) B. The diagnosis of cystic fibrosis is made only after an elevated sweat chloride test or demonstration of a genotype consistent with cystic fibrosis. (u) C. While recurrent respiratory infections is a classic presentation of cystic fibrosis, the diagnosis relies on confirmation, as noted in explanation B. (u) D. Trypsinogen levels are used as a neonatal screening test and if elevated should be followed by more definitive testing to confirm the diagnosis.

Scientific Concepts/Pulmonology Which of the following pathophysiological processes is associated with chronic bronchitis? A. Destruction of the lung parenchyma B. Mucous gland enlargement and goblet cell hyperplasia C. Smooth muscle hypertrophy in the large airways D. Increased mucus adhesion secondary to reduction in the salt and water content of the mucus

Explanations (u) A. Destruction of the gas-exchanging structures in the lung is characteristic of emphysema. (c) B. Chronic bronchitis results from the enlargement of mucous glands and goblet cell hypertrophy in the large airways. (u) C. There may be smooth muscle hypertrophy in chronic bronchitis but it is not to the extent as found in asthma and is not an underlying factor in the pathology of chronic bronchitis. (u) D. Abnormal absorption of sodium and a reduced rate of chloride secretion in cystic fibrosis leads to thickening of the mucus and increase in adhesion of the mucus.

Clinical Therapeutics/Pulmonology A 36 year-old male developed a sore throat and was treated with IM penicillin. Within 20 minutes, he felt faint, became dyspneic, and had diarrhea. Upon entry to the emergency department, he was pale and apprehensive. He had a thready pulse, and systolic blood pressure was 40 mmHg. Which of the following is the most appropriate initial agent to use? Answers A. Dopamine B. Epinephrine C. Hydrocortisone D. Diphenhydramine

Explanations (u) A. Dopamine is not indicated in the treatment of allergic reactions. (c) B. Epinephrine is the drug of first choice for emergency use and should be given as soon as anaphylactic shock is suspected or diagnosed. (u) C. Hydrocortisone should be given as an adjunct to epinephrine, but it is not the drug of first choice. (u) D. Diphenhydramine should be given as an adjunct to treatment, but it is not the drug of first choice.

History & Physical/Pulmonology Which of the following clinical manifestations is most commonly seen in viral croup? A. drooling B. wheezing C. sputum production D. inspiratory stridor

Explanations (u) A. Drooling is common in epiglottitis not viral croup. (u) B. Wheezing is noted in asthma. (u) C. Sputum production is noted in bacterial infections. (c) D. Viral croup typically presents with a barking cough and stridor.

History & Physical/Pulmonology The finding of egophony is most consistent with A. emphysema. B. atelectasis. C. pneumothorax. D. lobar pneumonia.

Explanations (u) A. Emphysema presents with diminished or absent breath sounds and hyperresonance to percussion without egophony. (u) B. Atelectasis most commonly has decreased breath sounds and dullness to percussion without egophony. (u) C. Pneumothorax presents with absent breath sounds, tactile fremitus, and resonance to percussion without egophony. (c) D. Egophony occurs with consolidation caused by lobar pneumonia.

Clinical Intervention/Pulmonology A 62 year-old male presents with a history of dyspnea on exertion and chronic cough worse with arising in the mornings. He has a 40-year-pack history of cigarette use. On examination there is increased AP diameter and decreased breath sounds with a prolonged expiratory phase. Pulse oximetry reveals an oxygen saturation of 93% on room air. In addition to smoking cessation, which of the following is an appropriate intervention at this time? Answers A. Home oxygen therapy B. Maintenance oral steroids C. Prophylactic antibiotic therapy D. Recommend influenza and pneumococcal vaccines

Explanations (u) A. Home oxygen therapy is indicated in COPD patients with an oxygen saturation < or equal to 88% or a pO2 < or equal to 55 mm Hg taken at rest breathing room air. (u) B. While oral steroids may be utilized in treatment of COPD, they are usually reserved for end stage disease due to the multiple systemic side effects of prolonged use. In addition only about 10% of patients show any increase in FEV1 and there use should be reserved for patients who show a 20% or greater improvement in FEV1. (u) C. Use of antibiotics should be reserved for treatment of acute exacerbations of COPD, acute bronchitis or documented bacterial infections, not prophylaxis. (c) D. In addition to smoking cessation, patients may benefit from vaccination against both influenza and pneumococcal disease.

Diagnosis/Pulmonology Which of the following physical examination findings would be consistent with a pleural effusion? A. Hyperresonance to percussion B. Increased tactile fremitus C. Unilateral lag on chest expansion D. Egophony

Explanations (u) A. Hyperresonance to percussion would be suggestive of emphysema or pneumothorax. (u) B. Increased tactile fremitus would be consistent with a consolidation. (c) C. A lag on chest expansion may be seen in the presence of a pleural effusion. (u) D. The presence of egophony would be consistent with a consolidation.

Health Maintenance/Pulmonology Which of the following is an independent risk factor for development of a mesothelioma? A. Cigarette smoking B. Asbestos exposure C. Radon gas exposure D. Chronic obstructive lung disease

Explanations (u) A. There has not been any evidence of association between cigarette smoking and the development of mesothelioma. (c) B. Studies confirm the association of asbestos exposure to the development of mesothelioma. (u) C. After cigarette smoking, radon gas is the second most common risk factor for development of bronchogenic lung cancer, not mesothelioma. (u) D. Chronic obstructive lung disease is associated with an increased risk of bronchogenic lung cancer, not mesothelioma.

Clinical Therapeutics/Pulmonology A 40 year-old alcoholic male presents with sudden onset of severe chills, fever, dyspnea and cough productive of red mucoid sticky sputum. He appears ill looking with cyanosis. Examination reveals vital signs: Temp - 102 degrees F; Pulse - 120 /minute and regular; 89 RR - 28/min; BP 90/62 mm Hg. Lungs reveal minimal rales and dullness in the right upper lobe with decreased breath sounds. Chest x-ray reveals right upper lobe consolidation with a bulging fissure. Gram stain reveals many white blood cells and many gram- negative rods. Which of the following is the most appropriate drug of choice? Answers A. Ticarcillin B. Cefotaxime C. Doxycycline D. Clindamycin

Explanations (u) A. Infections caused by Klebsiella organisms are resistant to both ticarcillin and ampicillin. (c) B. This patient most likely has pneumonia caused by Klebsiella. A third generation cephalosporin, such as cefotaxime, is the preferred antimicrobial therapy against Klebsiella pneumoniae. Alternative antibiotic choices may include a carbapenem, beta-lactam/beta- lactamase inhibitor or a fluoroquinolone. (u) C. See B for explanation. (u) D. See B for explanation.

Clinical Intervention/Pulmonology An elderly patient with poorly-controlled Type 2 diabetes and renal disease develops a fever of 102°F orally, productive cough, and dyspnea. Physical examination demonstrates a respiratory rate of 32/min, labored breathing, and rales at the left base. Pulse oximetry is 90%. Which of the following is the next appropriate step in the management of this patient? A. Administer nebulized corticosteroids B. Admit to the hospital C. Oral antimicrobial therapy D. Endotracheal intubation

Explanations (u) A. Inhaled corticosteroids are not utilized in the management of community-acquired pneumonia. (c) B. Community acquired pneumonia is the most deadly infectious disease in the U.S. Important risk factors for increased morbidity and mortality include advanced age, alcoholism, comorbid medical conditions, altered mental status, respiratory rate greater than 30 breaths/min, hypotension, and a BUN greater than 30. (u) C. Due to the age of the patient, comorbid diseases, and current signs of respiratory distress, intravenous not oral antimicrobial therapy is indicated. (u) D. Endotracheal intubation is indicated for respiratory failure unresponsive to conservative management.

Scientific Concepts/Pulmonology Which of the following best describes the pathophysiology of emphysema? Answers A. Interstitial inflammation and fibrosis B. Alveoli enlargement and loss of septa C. Mucosal edema and inflammatory response D. Excessive mucus secretion and chronic cough

Explanations (u) A. Interstitial inflammation and fibrosis are seen with restrictive causes of lung disease, such as asbestosis. (c) B. Emphysema results from alveoli enlargement with loss of septal wall integrity without any evidence of fibrosis. (u) C. Mucosal edema and inflammatory response are seen with asthma. (u) D. Excessive mucus secretion and chronic cough are characteristic of chronic bronchitis.

Clinical Therapeutics/Pulmonology A 2 year-old presents to the emergency department in acute respiratory distress. The parents relate a history of a recent upper respiratory illness that was followed by a sudden onset of barking cough during the night, but this morning they noted increased difficulty breathing. The child is noted to have stridor at rest, but has no evidence of cyanosis. Which of the following is the most appropriate initial intervention? A. Intravenous antibiotics B. Endotracheal intubation C. Inhaled mucolytic agent D. Nebulized racemic epinephrine

Explanations (u) A. Laryngotracheobronchitis is caused by viruses, not bacteria, and therefore antibiotic therapy is not indicated. (u) B. If patients fail to respond to initial treatment and progress to impending respiratory failure, endotracheal intubation is then indicated. (u) C. Inhaled mucolytic agents are not indicated in the treatment of laryngotracheobronchitis. (c) D. This patient most likely has laryngotracheobronchitis (viral croup). Treatment with nebulized racemic epinephrine and glucocorticosteroids is indicated for patients with stridor at rest.

History & Physical/Pulmonology Which of the following is a common presenting clinical manifestation of a patient with interstitial lung disease? A. Early inspiratory crackles B. Progressive dyspnea on exertion C. Productive cough with copious sputum D. Decreased breath sounds with hyperresonant percussion

Explanations (u) A. Late, not early, inspiratory crackles are associated with interstitial lung disease. (c) B. Patients with interstitial lung disease commonly present with progressive dyspnea on exertion and a cough with minimal sputum production. (u) C. A productive cough of copious amounts of sputum is most typical of a patient with chronic bronchitis. (u) D. Physical examination findings of decreased breath sounds with hyperresonant percussion is consistent with a diagnosis of chronic obstructive lung disease.

Scientific Concepts/Pulmonology The most common pathogen identified in community acquired pneumonia (CAP) is A. Mycoplasma pneumoniae. B. Staphylococcus aureus. C. Legionella pneumophila. D. Streptococcus pneumoniae.

Explanations (u) A. M. pneumoniae, S. aureus, Legionella pneumophila are common pathogens for CAP, but do not account for a majority of the cases. (u) B. See A for explanation. (u) C. See A for explanation. (c) D. S. pneumoniae accounts for a majority of cases of community acquired pneumonia.

Diagnosis/Pulmonology A 79 year-old female presents with productive cough for 2 days. She has associated fever, chills and shortness of breath. On physical exam, RR 30, BP 90/60, T 101.3. There is no JVD. Lungs reveal crackles at the left lower lobe and decreased breath sounds with dullness to percussion. Heart exam reveals RRR with no S3 or S4. No edema is noted. On chest x-ray the patient has a left sided pleural effusion. Examination of the pleural fluid reveals a decreased glucose and an elevated pleural fluid LDH. Pleural fluid cytology reveals squamous epithelial cells. What is the most likely cause of the patient's effusion? A. malignancy B. bacterial pneumonia C. heart failure D. pulmonary embolus

Explanations (u) A. Malignancy is also a leading cause of an exudative pleural effusion, second to bacterial pneumonia. Cytology in this case was normal. (c) B. Bacterial pneumonia is the leading cause of an exudative pleural effusion. (u) C. Pleural effusions in heart failure are transudative, not exudative pleural effusions. (u) D. PE can be both an exudative and transudative classification of effusion, however, dyspnea is usually the most common symptom.

Clinical Therapeutics/Pulmonology A 25 year-old male with a history of asthma presents complaining of increasing episodes of evening and daytime symptoms. He is on a short acting inhaled beta agonist prn. He is presently using his short acting beta agonist on a daily basis. Which of the following is the most appropriate addition to this patient's regimen? A. methylxanthine oxidase inhibitor B. long acting beta agonist inhaler C. leukotriene inhibitor D. inhaled corticosteroid

Explanations (u) A. Methylxanthine oxidase inhibitor preparations may have beneficial effects in some patients, but their value is limited due to a narrow therapeutic window and modest efficacy. (u) B. Long acting beta agonist inhalers should not be used in place of anti-inflammatory therapy. (u) C. Leukotriene inhibitors are less desirable alternatives to inhaled corticosteroids. (c) D. According to the stepwise approach for managing asthma by the National Asthma Education and Prevention Program, inhaled corticosteroids are indicated for mild to moderate persistent asthma.

Diagnosis/Pulmonology A 15 year-old male was seen last week with complaints of sore throat, headache, and mild cough. A diagnosis of URI was made and supportive treatment was initiated. He returns today with complaints of worsening cough and increasing fatigue. At this time, chest x-ray reveals bilateral hilar infiltrates. A WBC count is normal and a cold hemagglutinin titer is elevated. The most likely diagnosis is A. tuberculosis. B. mycoplasma pneumonia. C. pneumococcal pneumonia. D. staphylococcal pneumonia.

Explanations (u) A. Most children with pulmonary tuberculosis are asymptomatic with few physical examination findings. The results of the diagnostic studies do not support tuberculosis as the most likely diagnosis. (c) B. The insidious onset of symptoms, the interstitial infiltrate on chest x-ray, and elevated cold hemagglutinin titer make this diagnosis the most likely. (u) C. The clinical presentation of bacterial pneumonias in children is variable, but usually involves fever of acute onset. The WBC count is also usually elevated, making this a less likely diagnosis. (u) D. See C for explanation.

Diagnosis/Pulmonology A 32 year-old African American female presents with complaints of a gradual worsening of exertional dyspnea associated with a mild dry cough. She has tried various cough preparations on her own without any significant relief. Her examination is essentially unremarkable. A chest x-ray reveals the presence of bilateral hilar adenopathy. Which of the following is the most likely diagnosis? Answers A. silicosis B. sarcoidosis C. tuberculosis D. mycoplasma pneumonia

Explanations (u) A. Most patients with silicosis are asymptomatic, but in late stages it may present with dyspnea. A chest x-ray finding highly suggestive of silicosis is the calcification of the periphery of the hilar lymph nodes ("eggshell" calcification). (c) B. Patients with sarcoidosis present with an insidious onset of dyspnea that may be associated with malaise and fever. Incidence is the highest in the African American population and females are affected more frequently than males. Typical chest x-ray findings include bilateral hilar and right paratracheal lymphadenopathy. (u) C. The most common pulmonary complaint of tuberculosis is chronic cough associated with fatigue, weight loss, fever and night sweats. While dyspnea may be present it is a sign of extensive disease. In addition to hilar lymphadenopathy on chest x-ray, primary tuberculosis would also reveal small homogeneous infiltrates and segmental atelectasis. (u) D. While development of mycoplasma pneumonia is gradual, symptoms commonly include not only cough and dyspnea, but also fever, headache and sore throat. On exam most patients will also have rales and wheezes. A chest x-ray reveals diffuse interstitial infiltrates.

Clinical Therapeutics/Pulmonology An elderly nursing home resident is admitted with methicillin-resistant Staphylococcus aureus pneumonia. Which of the following is the most appropriate treatment to initiate? A. Nafcillin B. Vancomycin C. Clindamycin D. Doxycycline

Explanations (u) A. Nafcillin is classified as a penicillinase-resistant penicillin, however would not be effective against methicillin- resistant strains of Staphylococcus aureus. (c) B. Vancomycin with or without the addition of gentamicin or rifampin and linezolid is the treatment of choice for methicillin-resistant Staphylococcus aureus. (u) C. Clindamycin is primarily utilized in treatment of severe anaerobic infections, but is not indicated for the treatment of methicillin-resistant Staphylococcus aureus. (u) D. Doxycycline is a tetracycline and is only utilized to treat respiratory infections that are susceptible to tetracycline, such as Mycoplasma or Chlamydial pneumonias.

Clinical Therapeutics/Pulmonology A patient taking bleomycin (Blenoxane) should be monitored for which of the following side effects? A.Optic neuritis B. Hyperuricemia C. Encephalopathy D. Pulmonary fibrosis

Explanations (u) A. Optic neuritis is a potential side effect of ethambutol, used in the treatment of tuberculosis. (u) B. Hyperuricemia or encephalopathy are not known side effects of bleomycin. (u) C. See B for explanation. (c) D. Pulmonary fibrosis and pulmonary infiltrates are known side effects of bleomycin.

Clinical Therapeutics/Pulmonology A 22 year-old female with a history of asthma presents with complaints of increasing "asthma" attacks. The patient states she has been well controlled on albuterol inhaler until one month ago. Since that time she notices that she has had to use her inhaler 3-4 times a week and also has had increasing nighttime use averaging about three episodes in the past month. Spirometry reveals > 85% predicted value. Which of the following is the most appropriate intervention at this time? Answers A. Oral prednisone B. Oral theophylline C. Salmeterol inhaler D. Beclomethasone inhaler

Explanations (u) A. Oral corticosteroids, such as prednisone, are added to therapy in severe persistent asthma. While a course of oral corticosteroids may be needed for mild exacerbations of asthma, they are not added until inhaled corticosteroids have failed to control the symptoms. (u) B. Due to its safety profile, oral theophylline is now considered a third or fourth line treatment option for asthma. (u) C. Long acting inhaled beta2-agonists, such as salmeterol, are not added to the treatment regimen until the symptoms indicate a moderate persistent asthma. Long acting inhaled beta2 - agonists should also not be used in place of inhaled steroids. (c) D. This patient has progressed to mild persistent asthma. In addition to her inhaled beta2- agonist (albuterol), she should be started on an anti-inflammatory agent. Inhaled corticosteroids, such as beclomethasone, are preferred for long-term control. Other options may include cromolyn or nedocromil.

Clinical Therapeutics/Pulmonology A 34 year-old female with a history of asthma presents with complaints of increasing asthma attacks. The patient states she has been well-controlled on albuterol inhaler until one month ago. Since that time she notices that she has had to use her inhaler 3-4 times a week and also has had increasing nighttime use averaging about three episodes in the past month. Spirometry reveals greater than 85% predicted value. Which of the following is the most appropriate intervention at this time? A. Oral prednisone B. Oral theophylline (Theo-Dur) C. Salmeterol (Serevent) inhaler D. Beclomethasone (Qvar)inhaler

Explanations (u) A. Oral corticosteroids, such as prednisone, are added to therapy in severe persistent asthma. While a course of oral corticosteroids may be needed for mild exacerbations of asthma, they are not added until inhaled corticosteroids have failed to control the symptoms. (u) B. Due to its safety profile, oral theophylline is now considered a third or fourth line treatment option for asthma. (u) C. Long acting inhaled beta2-agonists, such as salmeterol, are not added to the treatment regimen until the symptoms indicate a moderate persistent asthma. Long acting inhaled beta2-agonists should also not be used in place of inhaled steroids. (c) D. This patient has progressed to mild persistent asthma. In addition to her inhaled beta2-agonist (albuterol), she should be started on an anti-inflammatory agent. Inhaled corticosteroids, such as beclomethasone, are preferred for long-term control.

Scientific Concepts/Pulmonology Which of the following mechanisms leads to a primary pneumothorax? A. Penetrating or blunt trauma forces B. Underlying lung cancer C. Pressure of air in the pleural space exceeds room air pressure D. Rupture of subpleural apical blebs due to high negative intrapleural pressures

Explanations (u) A. Penetrating or blunt trauma force is responsible for a traumatic pneumothorax. (u) B. A pneumothorax that results from an underlying lung disease is classified as a secondary pneumothorax. (u) C. When pressure of air in the pleural space exceeds room air pressure, it leads to a tension pneumothorax. (c) D. A primary spontaneous pneumothorax is thought to result from a rupture of subpleural apical blebs secondary to high negative intrapleural pressures.

Scientific Concepts/Pulmonology Which of the following mechanisms leads to a primary pneumothorax? Answers A. Penetrating or blunt trauma forces B. Underlying lung cancer C. Pressure of air in the pleural space exceeds room air pressure D. Rupture of subpleural apical blebs due to high negative intrapleural pressures

Explanations (u) A. Penetrating or blunt trauma force is responsible for a traumatic pneumothorax. (u) B. A pneumothorax that results from an underlying lung disease is classified as a secondary pneumothorax. (u) C. When pressure of air in the pleural space exceeds room air pressure, it leads to a tension pneumothorax. (c) D. A primary spontaneous pneumothorax is thought to result from a rupture of subpleural apical blebs secondary to high negative intrapleural pressures.

History & Physical/Pulmonology Which of the following is a physical examination finding that is consistent with a diagnosis of lobar pneumonia? A. Resonant to percussion B. Late inspiratory crackles C. Decreased tactile fremitus D. Tracheal shift toward uninvolved side

Explanations (u) A. Physical examination findings that are consistent with a diagnosis of lobar pneumonia include dullness to percussion, increased tactile fremitus and a trachea that is midline. (c) B. Late inspiratory crackles are a physical examination finding that is consistent with lobar pneumonia. (u) C. See A for explanation. (u) D. See A for explanation.

Diagnostic Studies/Pulmonology A 64 year-old female with a 50 pack year smoking history, presents with worsening dyspnea on exertion, a persistent cough, and increasing oxygen requirement from 2 to 3 liters. She denies any cardiac history. What is the most likely chest x-ray finding in this patient? A. pulmonary vascular congestion B. left lower lobe infiltrate C. apical infiltrates D. hyperinflation with bullae

Explanations (u) A. Pulmonary vascular congestion represents congestive heart failure not COPD. (u) B. Left lower lobe infiltrate represent an infectious process, such as pneumonia. (u) C. Apical infiltrates represent an infectious process, such as tuberculosis. (c) D. Hyperinflation with bullae is a consistent finding in patients with emphysema, such as this patient.

Diagnostic Studies/Pulmonology Which of the following is the minimum criteria in millimeter diameter of induration for a positive tuberculosis skin-test screening result in an HIV-positive patient? A. 0 B. 5 C. 10 D. 15

Explanations (u) A. See B for explanation. (c) B. Current minimum criteria for a positive skin test is 5 mm in diameter for individuals at very high risk, including those who are HIV infected and recent contacts. (u) C. See B for explanation. (u) D. See B for explanation.

Health Maintenance/Pulmonology A 65 year-old with COPD receiving their first pneumococcal conjugate vaccination should be revaccinated in A. 1 year. B. 3 years. C. 5 years. D. 10 years.

Explanations (u) A. See C for explanation. (u) B. See C for explanation. (c) C. A single revaccination for a person over the age of 65 is recommended if it has been more than 5 years since they received their first vaccination. (u) D. See C for explanation.

Clinical Therapeutics/Pulmonology Which of the following will result in decreased serum theophylline levels in a patient with COPD? A. Cimetidine B. Congestive heart failure C. Cigarette smoking D. Ciprofloxacin

Explanations (u) A. See C for explanation. (u) B. See C for explanation. (c) C. Cigarette smoking will increase the hepatic clearance of theophylline, resulting in decreased levels in the system. The use of cimetidine or ciprofloxacin or the presence of congestive heart failure will reduce hepatic clearance and causing an increase in theophylline serum levels. (u) D. See C for explanation.

Clinical Therapeutics/Pulmonology Which of the following is the most appropriate therapeutic agent for acute influenza? A. azithromycin (Zithromax) B. acyclovir (Zovirax) C. tetracycline (Sumycin) D. zanamivir (Relenza)

Explanations (u) A. See D for explanation. (u) B. See D for explanation. (u) C. See D for explanation. (c) D. Zanamivir is an anti-viral agent that is active against the influenza virus.

Health Maintenance/Pulmonology Which of the following forms of lung cancer is associated with the poorest prognosis? A. squamous cell B. adenocarcinoma C. large cell D. small cell

Explanations (u) A. See D for explanation. (u) B. See D for explanation. (u) C. See d for explanation. (c) D. Small cell lung cancer is the most common type of lung cancer that is metastatic at the time of discovery, and therefore has the poorest prognosis.

Diagnosis/Pulmonology A 53 year-old female status post abdominal hysterectomy 3 days ago suddenly develops pleuritic chest pain and dyspnea. On exam, she is tachycardic and tachypneic with rales in the left lower lobe. A chest x-ray is unremarkable and an EKG reveals sinus tachycardia. Which of the following is the most likely diagnosis? A. Atelectasis B. Pneumothorax C. Pulmonary embolism D. Myocardial infarction

Explanations (u) A. Small atelectasis is commonly asymptomatic, while large atelectasis may produce signs of dyspnea and cough. Exam reveals absence of breath sounds in the area involved and dullness to percussion. A chest x-ray would reveal various findings dependent on the location of the atelectasis, but would not be normal. (u) B. While a pneumothorax commonly presents with pleuritic chest pain and dyspnea, exam would reveal the presence of diminished breath sounds and hyperresonance on the involved side. A chest x-ray would reveal presence of a pleural line on the expiratory chest x-ray. (c) C. This patient's risk factors for pulmonary embolism include advanced age, surgery, and prolonged bedrest. While the diagnosis of pulmonary embolism is difficult to make due to nonspecific clinical findings, the most common symptoms include pleuritic chest pain and dyspnea associated with tachypnea. Chest x-ray and EKG are usually normal. (u) D. While a myocardial infarction usually presents with dyspnea, the chest pain is not usually pleuritic in nature. An EKG would commonly reveal ST segment changes which would be consistent with ischemia or infarct.

Clinical Intervention/Pulmonology A 62 year-old male presents with a right hilar mass. Needle-biopsy of the mass reveals the presence of small-cell carcinoma and a bone scan reveals the presence of scattered hot spots throughout the skeleton. Which of the following is the most appropriate treatment? A. Lobectomy B. Pneumonectomy C. Thoracic radiation therapy D. Combination chemotherapy

Explanations (u) A. Small-cell carcinoma of the lung is rarely treatable with surgical resection. Surgery may be indicated as part of the treatment protocol for small peripheral lesions without any evidence of metastasis. (u) B. See A for explanation. (u) C. While thoracic radiation therapy has been shown to be beneficial for patients with limited small-cell lung cancer, no benefit has been observed for patients with extensive disease defined as the presence of metastatic disease. (c) D. Combination chemotherapy is the treatment of choice for a patient with small-cell carcinoma of the lung.

Clinical Intervention/Pulmonology A 75 year-old man with a long history of COPD presents with acute onset of worsening dyspnea, increased productive cough, and marked agitation. While in the emergency department he becomes lethargic and obtunded. His ABG's reveal a PaO2 40 mmHg, PaCO2 65 mmHg, and arterial pH 7.25. Which of the following is the most appropriate management at this point? A. oxygen supplementation with a 100% non-rebreather mask B. noninvasive positive pressure ventilation (NIPPV) C. endotracheal intubation and mechanical ventilation D. emergency tracheostomy

Explanations (u) A. Supplemental oxygen and positive pressure ventilation are inadequate for patients with overt respiratory failure. (u) B. See A for explanation. (c) C. This patient is in severe respiratory arrest with markedly impaired mental status; conventional mechanical ventilation is required. (h) D. Tracheostomy is indicated for an obstructed airway.

Clinical Intervention/Pulmonology A 68 year-old male with history of COPD is brought to the emergency department following a motor vehicle collision. On physical examination there is evidence of head trauma. The left side of the chest wall appears to move inward with inspiration and outward with expiration. A chest x-ray reveals multiple rib fractures on the left. Which of the following is the most appropriate intervention? A. Surgical fixation of the fractured ribs B. Application of elastic binders and adhesive tape C. Endotracheal intubation and mechanical ventilation D. Chest physiotherapy that encourages frequent coughing

Explanations (u) A. Surgical fixation of flail chest is less reliable than positive-pressure ventilation and is performed only rarely in the United States. (u) B. While application of elastic binders and adhesive tape was historically utilized to stabile the chest, this intervention has been found to decrease chest expansion and worsen lung atelectasis. (c) C. Indications for early endotracheal intubation and mechanical ventilation in treatment of flail chest include patients that are over the age of 65, have comorbid lung disease and associated severe head trauma. Other indications include shock, three or more associated injuries and fracture of eight or more ribs. (u) D. Conservative treatment for mild to moderate flail chest includes analgesic relief of pain, chest physiotherapy that encourages frequent coughing and restriction of fluids to prevent fluid overload, however this patient needs early ventilatory support.

Clinical Therapeutics/Pulmonology A 26 year-old man is stung by a bee, and shortly thereafter, a wheal develops at the site of the sting. He soon feels flushed and develops hives, rhinorrhea, and tightness in the chest. He is seen in the urgent care center. Immediate therapy should be to A. transfer him to a local hospital emergency department. B. apply a cold compress to site of the sting. C. administer subcutaneous epinephrine. D. administer oral albuterol.

Explanations (u) A. Systemic (anaphylactic) reactions can rapidly become life-threatening. Delay in treatment may cause death. (u) B. This is only supportive local therapy and does not address the need to treat the systemic reaction present. (c) C. Epinephrine hydrochloride 1:1000, 0.2 to 0.5 mL subcutaneously is indicated for the initial treatment of this systemic reaction. Additional injections may be given every 20 to 30 minutes if needed. (u) D. Albuterol is indicated in the presence of bronchospasm (suggested by the presence of chest tightness), but would be delivered by an aerosol, not an oral, route.

Clinical Therapeutics/Pulmonology An immunocompromised patient presents with signs and symptoms consistent with Legionella pneumophila who has not responded to initial antibiotic therapy with a macrolide. Which of the following should be added? A. Clarithromycin (Biaxin) B. Rifampin (Rifadin) C. Levofloxacin (Levaquin) D. Amoxicillin-clavulanate (Augmentin)

Explanations (u) A. The macrolides (Clarithromycin) and fluoroquinolones (Levofloxacin) should be used for initial treatment, but not for adding to failed treatments when a macrolide was already used. (c) B. Rifampin should be used as an adjunct in patients with either a macrolide or quinolone antibiotic, who have failed therapy, are immunocompromised or have severe illness. (u) C. See A for explanation. (u) D. Legionella pneumophila does not respond to Beta-lactam antibiotics.

Health Maintenance/Pulmonology Which of the following is an independent risk factor for development of a mesothelioma? Answers A. Cigarette smoking B. Asbestos exposure C. Radon gas exposure D. Chronic obstructive lung disease

Explanations (u) A. There has not been any evidence of association between cigarette smoking and the development of mesothelioma. (c) B. Studies confirm the association of asbestos exposure to the development of mesothelioma. (u) C. After cigarette smoking, radon gas is the second most common risk factor for development of bronchogenic lung cancer, not mesothelioma. (u) D. Chronic obstructive lung disease is associated with an increased risk of bronchogenic lung cancer, not mesothelioma.

Diagnostic Studies/Pulmonology A 73 year-old obese female with a 20 pack year smoking history presents complaining of chronic productive cough. She states that it has been occurring over the past 3 years more frequently November through February. Which of the following pulmonary function test values would you expect to find decreased? A. tidal volume B. forced expiratory volume in 1 second/forced vital capacity C. residual volume D. total lung capacity

Explanations (u) A. Tidal volume is usually unchanged, residual volume and total lung capacity are decreased with a restrictive disease pattern. This scenario presents with bronchitis which is an obstructive disease. (c) B. Forced expiratory volume in 1 second/forced vital capacity is decreased in obstructive lung diseases such as bronchitis. (u) C. See A for explanation. (u) D. See A for explanation.

Diagnostic Studies/Pulmonology A post-op patient has signs and symptoms highly suggestive of a pulmonary embolism. The results of the CT scan of the lung is nondiagnostic. What is the most appropriate next step in the evaluation? A. Ventilation perfusion (V/Q) scan B. Ultrasound of the legs C. Echocardiography D. D-dimer

Explanations (u) A. Ventilation perfusion scans are performed prior to the CT scan of the chest and would not likely add additional information to this clinical scenario. (c) B. In a patient with a high likelihood of pulmonary embolism or an inpatient, as in this case, ultrasound of the legs would be the next diagnostic step after a nondiagnostic CT. (u) C. Although echocardiography may show right ventricular free wall hypokinesis with normal motion of the apex suggestive of pulmonary embolism, more than 50% of patients with a pulmonary embolism will have normal echocardiography. Echocardiography is not used in the diagnosis on inpatients.; (u) D. In a post-op patient, a d-dimer will be positive regardless of the presence or absence of a pulmonary embolism.

cientific Concepts/Pulmonology Which of the following has been found to be an important reservoir in the transmission of pertussis? Answers A. Mosquitoes B. Adult humans C. Dogs and cats D. White-tailed deer

Explanations (u) A. While mosquitoes have been implemented in the spread of several infectious diseases, pertussis is not one of them. (c) B. Since neither the vaccine nor the disease itself provides lasting immunity against pertussis, adult humans are an important reservoir in the transmission of pertussis. (u) C. Contact with dogs and cats has lead to the development of upper and lower respiratory infections that are caused by Bordetella bronchiseptica, but not pertussis. (u) D. White-tailed deer are part of the transmission cycle for Lyme disease, not pertussis.

Scientific Concepts/Pulmonology Which of the following has been found to be an important reservoir in the transmission of pertussis? A. mosquitoes B. adult humans C. domestic pets D. white-tailed deer

Explanations (u) A. While mosquitoes have been implemented in the spread of several infectious diseases, pertussis is not one of them. (c) B. Since neither the vaccine nor the disease itself provides lasting immunity against pertussis, adult humans are an important reservoir in the transmission of pertussis. (u) C. Contact with domestic pets has led to the development of upper and lower respiratory infections that are caused by Bordetella bronchiseptica, but not pertussis. (u) D. White-tailed deer are part of the transmission cycle for Lyme disease, not pertussis.

Diagnosis/Pulmonology You are called to the nursery to see a male infant, born by uncomplicated vaginal delivery. He weighs 2,600 grams and has one deep crease on the anterior third of each foot. Respirations are 88 breaths/minute with expiratory grunting and intercostals retractions. He is cyanotic on room air and becomes pink when placed on 60% oxygen. Chest x-ray shows atelectasis with air bronchograms. Which of the following is the most likely diagnosis? A. neonatal pneumonia B. congenital heart disease C. hyaline membrane disease D. chronic lung disease of prematurity

Explanations (u) A. While tachypnea, grunting, retractions and cyanosis may be signs of neonatal pneumonia, they are primarily late findings of progressive respiratory distress and would not be seen immediately at the time of delivery. A chest x- ray in pneumonia would also most commonly reveal an infiltrate or effusion. (u) B. While congenital heart disease may present with cyanosis, the chest x-ray will reveal a cardiac abnormality, such as cardiomegaly. (c) C. Hyaline membrane disease is the most common cause of respiratory distress in the premature infant. The infant typically presents with tachypnea, cyanosis and expiratory grunting. A chest x-ray reveals hypoexpansion and air bronchograms. (u) D. Chronic lung disease of prematurity is a complication in about 20% of infants with hyaline membrane disease. It is defined as respiratory symptoms, oxygen requirement and chest x-ray abnormalities at 1 month of age so it cannot be diagnosed at this time in this newborn.

Diagnosis/Pulmonology A 45 year-old male presents with complaints of a chronic cough productive of mucopurulent sputum. The cough has been present for the past 3 years, but he attributed it to a "smoker's cough". He has been coughing up a lot of sputum lasting all winter long for the past 2 years. He denies any hemoptysis, weight loss or chest pain. Physical examination reveals a moderately obese male in no acute respiratory distress. Lung fields reveal presence of scattered rhonchi and wheezes. There is 1+ peripheral edema. Which of the following is the most likely diagnosis? A. Lung cancer B. Bronchiectasis C. Chronic bronchitis D. Interstitial lung disease

Explanations (u) A. While the respiratory complaints of lung cancer are associated with the location and type of primary tumor, anorexia and weight loss is seen in the majority of patients. Patients will also usually have a new cough or a change in a chronic cough and may complain of hemoptysis and nonspecific chest pain. (u) B. While bronchiectasis presents with a chronic cough productive of copious amounts of purulent sputum, these patients most commonly also have associated complaints of hemoptysis, weight loss and pleuritic chest pain. Examination of the lungs reveals persistent crackles at the bases. (c) C. This patient most likely has chronic bronchitis which is defined as sputum production and cough for at least 3 months of the year for 2 consecutive years which is primarily caused by cigarette smoking. (u) D. Interstitial lung disease is characterized by progressive exertional dyspnea and cough, however sputum production is minimal and the examination of the lungs reveals fine, late inspiratory crackles at the bases in the majority of patients.


Related study sets

11.4 (paso 1:2) - AY POR DIOX it was an accident (reflexive useage)

View Set

Python 2.1 | The "Hello, World!" Program

View Set

CORPORALS COURSE COMP 2 TEST REVIEW

View Set

Chapter 12: Dimensions of Marketing Strategy

View Set

Neuromusculoskeletal System Exam I Content

View Set